Онлайн решение матриц методом гаусса: Онлайн калькулятор. Решение систем линейных уравнений. Метод Гаусса.

Содержание

Решение СЛАУ 4-ого порядка методом Гаусса, пример № 4

СЛАУ 3-его порядка: 1 — 2 — 3 — 4 — 5 — 6 — 7 — 8 — 9 — 10 — 11 — 12
СЛАУ 4-ого порядка: 1 — 2 — 3 — 4 — 5 — 6 — 7 — 8 — 9 — 10 — 11 — 12


Условие

 3x 1 + x 2 — 2x 3 — 2x 4   =   -2
 2x 1 — x 2 + 2x 3 + 2x 4   =   2
 2x 1 + x 2 — x 3 — x 4   =   -1
 x 1 + x 2 — 3x 3 + 2x 4   =   -3

Решение системы линейных алгебраических уравнений методом Гаусс

Для проверки ответов можете воспользоваться нашим онлайн сервисом — Решение системы линейных уравнений методом Гаусса. Все действия описанные в данном разделе не противоречат правилам обращения с матрицами и являются элементарными преобразованиями матрицы. Если после изучения примеров решения задач у Вас останутся вопросы, то Вы всегда можете задать их на форуме, и не забывайте про наши онлайн калькуляторы для решения задач по математике и другим предметам!

Перепишем систему линейных алгебраических уравнений в матричную форму. Получится матрица 4 × 5, слева от разделительной линии стоят коэффициенты при переменных, а справа стоят свободные члены.


Проведём следующие действия:

  • Поменяем местами строку № 1 и строку № 4

Получим:

Проведём следующие действия:

  • Из строки № 2 вычтем строку № 1 умноженную на 2 (Строка 2 — 2 × строка 1)
  • Из строки № 3 вычтем строку № 1 умноженную на 2 (Строка 3 — 2 × строка 1)
  • Из строки № 4 вычтем строку № 1 умноженную на 3 (Строка 4 — 3 × строка 1)

Получим:

Проведём следующие действия:

  • Строку № 3 умножим на -1 (Строка 3 = строка 3 * -1)
  • Поменяем местами строку № 2 и строку № 3

Получим:

Проведём следующие действия:

  • К строке № 3 прибавим строку № 2 умноженную на 3 (Строка 3 + 3 × строка 2)
  • К строке № 4 прибавим строку № 2 умноженную на 2 (Строка 4 + 2 × строка 2)

Получим:

Проведём следующие действия:

  • Строку № 4 поделим на -3 (Строка 4 = строка 4 / -3)
  • Поменяем местами строку № 3 и строку № 4

Получим:

Проведём следующие действия:

  • К строке № 4 прибавим строку № 3 умноженную на 7 (Строка 4 + 7 × строка 3)

Получим:

Проведём следующие действия:

  • Строку № 4 поделим на 55 (Строка 4 = строка 4 / 55)
  • Из строки № 3 вычтем строку № 4 умноженную на 6 (Строка 3 — 6 × строка 4)
  • Из строки № 2 вычтем строку № 4 умноженную на 5 (Строка 2 — 5 × строка 4)
  • Из строки № 1 вычтем строку № 4 умноженную на 2 (
    Строка 1 — 2 × строка 4
    )

Получим:

Проведём следующие действия:

  • К строке № 2 прибавим строку № 3 умноженную на 5 (Строка 2 + 5 × строка 3)
  • К строке № 1 прибавим строку № 3 умноженную на 3 (Строка 1 + 3 × строка 3)

Получим:

Проведём следующие действия:

  • Из строки № 1 вычтем строку № 2 (Строка 1 — строка 2)

Получим:

В левой части матрицы по главной диагонали остались одни единицы. В правом столбце получаем решение:
х1 = 0
х2 = 0
х3 = 1
х4 = 0


Вы поняли, как решать? Нет?

Помощь с решением

Матричный метод решения систем линейных уравнений

Матричный метод может применяться в решении систем линейных уравнений, в которых число неизвестных равно числу уравнений, то есть систем линейных уравнений с квадратной матрицей коэффициентов при неизвестных.

Другое условие применимости матричного метода — невырожденность матрицы коэффициентов при неизвестных, то есть неравенство нулю определителя этой матрицы.

Систему линейных уравнений, при выполнении вышеназванных условий, можно представить в матричном виде, а затем решить её путём отыскания обратной матрицы к матрице системы.

Решение систем линейных уравнений матричным методом основано на следующем свойстве обратной матрицы: произведение обратной матрицы и исходной матрицы равно единичной матрице. Обратная матрица обозначается символом .

Пусть нужно решить систему линейных уравнений:

Запишем эту систему уравнений в матричном виде:

Обозначим отдельно как A матрицу коэффициентов при неизвестных и как B матрицу неизвестных и матрицу свободных членов

.

Тогда

То есть, для нахождения решений системы нужно обе части уравнения умножить на матрицу, обратную матрице коэффициентов при неизвестных и приравнять соответствующие элементы полученных матриц.

Алгоритм решения системы линейных уравнений матричным методом разберём на следующем примере системы линейных уравнений второго порядка.

Пример 1. Решить матричным методом систему линейных уравнений:

Решение состоит из следующих шагов.

Шаг 1. Составляем следующие матрицы.

Матрица коэффициентов при неизвестных:

Матрица неизвестных:

Матрица свободных членов:

Это сделано для того, чтобы применить в решении уже записанные закономерности, основанные на свойстве обратной матрицы:

По выведенному выше последнему равенству и будем вычислять решения данной системы.

Но сначала проверим, не является ли матрица коэффициентов при неизвестных вырожденной, то есть можем ли вообще применять матричный метод:

.

Определитель этой матрицы не равен нулю, следовательно, можем применять матричный метод.

Шаг 2. Находим матрицу, обратную матрице коэффициентов при неизвестных:

.

Шаг 3. Находим матрицу неизвестных:

Итак, получили решение:

.

Сделаем проверку:

Следовательно, ответ правильный.

Для второго примера выберем систему линейных уравнений третьего порядка.

Пример 2. Решить матричным методом систему линейных уравнений:

Шаг 1. Составляем следующие матрицы.

Матрица коэффициентов при неизвестных:

Матрица неизвестных:

Матрица свободных членов:

Проверим, не является ли матрица коэффициентов при неизвестных вырожденной:

.

Определитель этой матрицы не равен нулю, следовательно, можем применять матричный метод.

Шаг 2. Находим матрицу, обратную матрице коэффициентов при неизвестных:

.

Шаг 3. Находим матрицу неизвестных:

Итак, получили решение:

.

Сделаем проверку:

Следовательно, ответ правильный.

Решить систему уравнений матричным методом самостоятельно, а затем посмотреть решение

Всё по теме «Системы уравнений и неравенств»

Начало темы «Линейная алгебра»

Поделиться с друзьями

Системы линейных уравнений. Метод Гаусса

1. ЛИНЕЙНАЯ АЛГЕБРА

Лекция 5
СИСТЕМЫ ЛИНЕЙНЫХ УРАВНЕНИЙ
(продолжение)
МЕТОД ГАУССА
§ 1. МЕТОД ГАУССА
Решить систему линейных уравнений – значит получить
равносильную ей систему, которая уже является разрешенной
или несовместной. Это удобно сделать при помощи метода
Гаусса, который позволяет привести систему к более простому
виду, с помощью элементарных преобразований строк в
расширенной матрице системы
Пусть дана система линейных уравнений. Поставим на
первое место любое уравнение с ненулевым коэффициентом при
x1:
• Шаг 1: умножим каждое уравнение, кроме первого, на
множитель a11/ai1, где i -номер уравнения в системе (номер
строки системы).
• после этого все коэффициенты при переменной x1 во всех
уравнениях равны a11.

Шаг 2: Вычтем из каждого уравнения системы, начиная
со второго, первое уравнение. Получим систему, в которой
все коэффициенты при x1 во всех уравнениях, кроме
первого обратились в ноль.
Повторить шаги 1-2 для второго столбца, начиная с третьего
уравнения. И т.д.
• Рассмотрим частные случаи приведенных по методу Гаусса
систем в случае с тремя неизвестными.

Случай 1. Система методом Гаусса приведена к следующему
виду:
В данном случае система имеет единственное решение,
которое получается последовательным нахождением
переменных, начиная с последнего уравнения:
Замечание: в данном случае ранг основной матрицы равен
3, ранг расширенной матрицы также равен 3.
• Случай 2. Система методом Гаусса приведена к следующему
виду:
• В данном случае система из-за последнего уравнения
несовместна и, следовательно, не имеет решений.
• Ранг основной матрицы системы очевидно равен 2.
• Рассмотрим расширенную матрицу системы и минор из первого
столбца, второго столбца и столбца свободных членов. Порядок
полученного минора равен 3.
• Следовательно, ранг расширенной матрицы больше ранга
матрицы системы.
• В этом случае система решения не имеет.
• Случай 3. Система методом Гаусса приведена к следующему
виду:
• Последнее уравнение системы обратилось в ноль, и
система стала недоопределенной – два уравнения на три
неизвестных. Запишем решение системы следующим
образом:
• Задавая различные значения параметра k, мы получим
различные решения системы. Следовательно, решений
бесконечно много. Так как решение зависит от одного
параметра, то размерность решения равна 1.
Рассмотрим ранги основной матрицы системы и расширенной
матрицы.
Они, очевидно, совпадают (равны 2), но меньше размерности
системы (количества неизвестных).
Теорема Кронекера-Капелли: Для того чтобы линейная
система являлась совместной, необходимо и достаточно,
чтобы ранг расширенной матрицы этой системы был равен
рангу ее основной матрицы.
Следствие: Если ранги основной и расширенной матриц
линейной системы совпадают с количеством переменных, то
система имеет единственное решение.
При применении метода Гаусса на практике следует производить
преобразования над строками расширенной матрицы системы.
Пример. Решить методом Гаусса систему
Решение. Расширенная матрица системы имеет вид
• Прибавив ко второй строке первую, умноженную на (-2), к
третьей – первую, умноженную на (-3), к четвертой – первую,
умноженную на (-1), получим
Разделим третью строку на 13 и поменяем местами вторую и
третью строки:
Прибавим к третьей строке вторую, умноженную на (-9), к
четвертой – вторую, умноженную на (-2):
Разделив вторую
строку на (-2), а
третью на (-7),
имеем
Этой матрице
соответствует
система
Осуществляя обратный ход, находим: x4 = −2, x3 = 2,
Полагая x2 = c, получаем общее решение:

Метод Жордана-Гаусса — онлайн калькулятор с подробным решением

Данное решение сделано калькулятором, представленным на сайте.

Пожалуйста, обратите внимание, что коэффициенты расположенные на «красных» позициях исчезают.

3x1+2x2+x3+x4 = — 2
x1x2+4x3x4 = — 1
2x1 2x2 3x3+x4 = 9
x1+5x2x3+2x4 = 4

Уравнения 1 и 2 поменяем местами.

x1x2+4x3x4 = — 1
3x1+2x2+x3+x4 = — 2
2x1 2x2 3x3+x4 = 9
x1+5x2x3+2x4 = 4
К уравнению 2 прибавляем уравнение 1, умноженное на -3.   подробнее

( 3 x1 + x1 * ( -3) )

+ ( 2 x2 + ( — x2) * ( -3) )

+ ( x3 + 4 x3 * ( -3) )

+ ( x4 + ( — x4) * ( -3) )

= -2 + ( -1) * ( -3)

«Красный» коэффициент равен нулю.

x1x2+4x3x4 = — 1
5x2 11x3+4x4 = 1
2x1 2x2 3x3+x4 = 9
x1+5x2x3+2x4 = 4
К уравнению 3 прибавляем уравнение 1, умноженное на 2.   подробнее

( -2 x1 + x1 * 2 )

+ ( -2 x2 + ( — x2) * 2 )

+ ( -3 x3 + 4 x3 * 2 )

+ ( x4 + ( — x4) * 2 )

= 9 + ( -1) * 2

«Красный» коэффициент равен нулю.

x1x2+4x3x4 = — 1
5x2 11x3+4x4 = 1
4x2+5x3x4 = 7
x1+5x2x3+2x4 = 4
К уравнению 4 прибавляем уравнение 1, умноженное на -1.   подробнее

( x1 + x1 * ( -1) )

+ ( 5 x2 + ( — x2) * ( -1) )

+ ( — x3 + 4 x3 * ( -1) )

+ ( 2 x4 + ( — x4) * ( -1) )

= 4 + ( -1) * ( -1)

«Красный» коэффициент равен нулю.

x1x2+4x3x4 = — 1
5x2 11x3+4x4 = 1
4x2+5x3x4 = 7
6x2 5x3+3x4 = 5

( 5 x2 + ( -4 x2) )

+ ( -11 x3 + 5 x3)

+ ( 4 x4 + ( — x4) )

= 1 + 7

Данное преобразование позволит нам считать без дробей какое то время.

x1x2+4x3x4 = — 1
x2 6x3+3x4 = 8
4x2+5x3x4 = 7
6x2 5x3+3x4 = 5
К уравнению 3 прибавляем уравнение 2, умноженное на 4.   подробнее

( -4 x2 + x2 * 4 )

+ ( 5 x3 + ( -6 x3) * 4 )

+ ( — x4 + 3 x4 * 4 )

= 7 + 8 * 4

«Красный» коэффициент равен нулю.

x1x2+4x3x4 = — 1
x2 6x3+3x4 = 8
19x3+11x4 = 39
6x2 5x3+3x4 = 5
К уравнению 4 прибавляем уравнение 2, умноженное на -6.   подробнее

( 6 x2 + x2 * ( -6) )

+ ( -5 x3 + ( -6 x3) * ( -6) )

+ ( 3 x4 + 3 x4 * ( -6) )

= 5 + 8 * ( -6)

«Красный» коэффициент равен нулю.

x1x2+4x3x4 = — 1
x2 6x3+3x4 = 8
19x3+11x4 = 39
31x3 15x4 = — 43
К уравнению 4 прибавляем уравнение 3, умноженное на 31/19.   подробнее

( 31 x3 + ( -19 x3) * 31/19 )

+ ( -15 x4 + 11 x4 * 31/19 )

= -43 + 39 * 31/19

«Красный» коэффициент равен нулю.

x1x2+4x3x4 = — 1
x2 6x3+3x4 = 8
19x3+11x4 = 39
56/19x4 = 392/19

Уравнеие 4 разделим на 56/19.

x1x2+4x3x4 = — 1
x2 6x3+3x4 = 8
19x3+11x4 = 39
x4 = 7
К уравнению 3 прибавляем уравнение 4, умноженное на -11.   подробнее

— 19 x3

+ ( 11 x4 + x4 * ( -11) )

= 39 + 7 * ( -11)

«Красный» коэффициент равен нулю.

x1x2+4x3x4 = — 1
x2 6x3+3x4 = 8
19x3 = — 38
x4 = 7
К уравнению 2 прибавляем уравнение 4, умноженное на -3.   подробнее

x2

— 6 x3

+ ( 3 x4 + x4 * ( -3) )

= 8 + 7 * ( -3)

«Красный» коэффициент равен нулю.

x1x2+4x3x4 = — 1
x2 6x3 = — 13
19x3 = — 38
x4 = 7

x1

+ — x2

+ 4 x3

+ ( — x4 + x4)

= -1 + 7

«Красный» коэффициент равен нулю.

x1x2+4x3 = 6
x2 6x3 = — 13
19x3 = — 38
x4 = 7

Уравнеие 3 разделим на -19.

x1x2+4x3 = 6
x2 6x3 = — 13
x3 = 2
x4 = 7
К уравнению 2 прибавляем уравнение 3, умноженное на 6.   подробнее

x2

+ ( -6 x3 + x3 * 6 )

= -13 + 2 * 6

«Красный» коэффициент равен нулю.

x1x2+4x3 = 6
x2 = — 1
x3 = 2
x4 = 7
К уравнению 1 прибавляем уравнение 3, умноженное на -4.   подробнее

x1

+ — x2

+ ( 4 x3 + x3 * ( -4) )

= 6 + 2 * ( -4)

«Красный» коэффициент равен нулю.

x1x2 = — 2
x2 = — 1
x3 = 2
x4 = 7

x1

+ ( — x2 + x2)

= -2 + ( -1)

«Красный» коэффициент равен нулю.

x1 = — 3
x2 = — 1
x3 = 2
x4 = 7

Ответ:

x1 = — 3

x2 = — 1

x3 = 2

x4 = 7

Решение высшей математики онлайн


‹— Назад


Алгоритм нахождения решений произвольной системы линейных уравнений (метод Гаусса) Пусть дана система линейных уравнений с неизвестными . Требуется найти ее общее решение, если она совместна, или установить ее несовместность. Метод, который будет изложен в этом разделе, близок к методу вычисления определителя 5.1.с и к методу нахождения ранга матрицы (раздел 5.8). Предлагаемый алгоритм называется методом Гаусса или методом последовательного исключения неизвестных.

Выпишем расширенную матрицу системы

Назовем элементарными операциями следующие действия с матрицами:

  1. перестановка строк;
  2. умножение строки на число, отличное от нуля;
  3. сложение строки с другой строкой, умноженной на число.

Отметим, что при решении системы уравнений, в отличие от вычисления определителя и нахождения ранга, нельзя оперировать со столбцами.

Читатель легко проверит, что если по матрице, полученной из выполнением элементарной операции, восстановить систему уравнений, то новая система будет равносильна исходной.

Цель алгоритма — с помощью применения последовательности элементарных операций к матрице добиться, чтобы каждая строка, кроме, быть может, первой, начиналась с нулей, и число нулей до первого ненулевого элемента в каждой следующей строке было больше, чем в предыдущей.

Шаг алгоритма заключается в следующем. Находим первый ненулевой столбец в матрице . Пусть это будет столбец с номером . Находим в нем ненулевой элемент и строку с этим элементом меняем местами с первой строкой. Чтобы не нагромождать дополнительных обозначений, будем считать, что такая смена строк в матрице уже произведена, то есть . Тогда ко второй строке прибавим первую, умноженную на число , к третьей строке прибавим первую, умноженную на число , и т.д. В результате получим матрицу

(Первые нулевые столбцы, как правило, отсутствуют.)

Если в матрице встретилась строка с номером , в которой все элементы равны нулю, а , то выполнение алгоритма останавливаем и делаем вывод, что система несовместна. Действительно, восстанавливая систему уравнений по расширенной матрице, получим, что -ое уравнение будет иметь вид

Этому уравнению не удовлетворяет ни один набор чисел .

Матрицу можно записать в виде

где По отношению к матрице выполняем описанный шаг алгоритма. Получаем матрицу где , . Эту матрицу снова можно записать в виде и к матрице снова применим описанный выше шаг алгоритма.

Процесс останавливается, если после выполнения очередного шага новая уменьшенная матрица состоит из одних нулей или если исчерпаны все строки. Заметим, что заключение о несовместности системы могло остановить процесс и ранее.

Если бы мы не уменьшали матрицу, то в итоге пришли бы к матрице вида

Далее выполняется так называемый обратный ход метода Гаусса. По матрице составляем систему уравнений. В левой части оставляем неизвестные с номерами, соответствующими первым ненулевым элементам в каждой строке, то есть . Заметим, что . Остальные неизвестные переносим в правую часть. Считая неизвестные в правой части некоторыми фиксированными величинами, несложно выразить через них неизвестные левой части.

Теперь, придавая неизвестным в правой части произвольные значения и вычисляя значения переменных левой части, мы будем находить различные решения исходной системы . Чтобы записать общее решение, нужно неизвестные в правой части обозначить в каком-либо порядке буквами , включая и те неизвестные, которые явно не выписаны в правой части из-за нулевых коэффициентов, и тогда столбец неизвестных можно записать в виде столбца, где каждый элемент будет линейной комбинацией произвольных величин (в частности, просто произвольной величиной ). Эта запись и будет общим решением системы.

Если система была однородной, то получим общее решение однородной системы. Коэффициенты при , взятые в каждом элементе столбца общего решения, составят первое решение из фундаментальной системы решений, коэффициенты при  — второе решение и т.д.

Фундаментальную систему решений однородной системы можно получить и другим способом. Для этого одному переменному, перенесенному в правую часть, нужно присвоить значение 1, а остальным — нули. Вычислив значения переменных в левой части, получим одно решение из фундаментальной системы. Присвоив другому переменному в правой части значение 1, а остальным — нули, получим второе решение из фундаментальной системы и т.д.

        Замечание 15.4   У читателя может возникнуть вопрос: «Зачем рассматривать случай, когда некоторые столбцы матрицы нулевые? Ведь в этом случае соответствующие им переменные в системе уравнений в явном виде отсутствуют.» Но дело том, что в некоторых задачах, например, при нахождении собственных чисел матрицы, такие системы возникают, и игнорировать отсутствующие переменные нельзя, так как при этом происходит потеря важных для задачи решений.                  Пример 15.2   Найдите общее решение системы уравнений где неизвестными являются .

Решение. Выпишем расширенную матрицу системы

Прибавим ко второй строке первую, умноженную на число , к третьей строке прибавим первую, умноженную на . В результате получим Прибавим к третьей строке вторую, умноженную на число . Получим Прямой ход метода Гаусса закончен. Выписываем по матрице систему уравнений Переносим в правую часть неизвестные (неизвестное реально в ней присутствовать не будет, коэффициент перед ним равен нулю). Получаем Пусть , , , . Из уравнений находим:

Ответ: , , , , , , где , , ,  — произвольные числа.         

        Замечание 15.5   В процессе решения можно также установить, какие ранги у матриц и и где расположены их базисные миноры. В предыдущем примере , базисный минор расположен в строках с номерами 1, 2, столбцах с номерами 2, 5.                  Пример 15.3   Найдите общее решение системы уравнений

Решение. Запишем расширенную матрицу системы:

Ко второй строке прибавим первую, умноженную на , к третьей строке прибавим первую, умноженную на , к четвертой строке прибавим первую, умноженную на : Вторую строку, умноженную на , прибавим к третьей: В третьей строке все элементы равны нулю, а элемент . Значит, система несовместна.

Ответ: Система несовместна.         

        Пример 15.4   Решите систему

Решение. Имеем:

Первую строку, умноженную на числа , , , прибавим соответственно ко второй, третьей и четвертой строкам: К третьей строке прибавим вторую, умноженную на . Получим К четвертой строке прибавим третью, умноженную на : Выписываем по матрице систему уравнений: Находим последовательно значения неизвестных:

Ответ: .         

        Замечание 15.6   Так же, как и при решении системы уравнений по правилу Крамера, при использовании метода Гаусса приходится выполнять большой объем вычислительной работы. Из-за этого вполне возможно, что будет допущена какая-либо ошибка в вычислениях. Поэтому желательно после решения системы выполнить проверку, то есть подставить полученные значения неизвестных в уравнения системы. Для выполнения полной проверки подстановку нужно произвести во все уравнения системы. Если же по каким-то причинам это не выполнимо, то можно подставить найденные значения в одно уравнение. В отличие от правила Крамера в методе Гаусса эту подстановку нужно производить в ПОСЛЕДНЕЕ уравнение исходной системы. При наличии в этом уравнении всех неизвестных эта подстановка почти всегда покажет наличие ошибки, если таковая была допущена.         

        Пример 15.5   Найдите фундаментальную систему решений и общее решение однородной системы линейных уравнений:

Решение. Составляем расширенную матрицу системы:

Умножим первую строку последовательно на , 5 и 1 и прибавим соответственно ко второй, третьей и четвертой строкам. Получим матрицу Вторую строку умножим последовательно на числа 4 и 2 и прибавим соответственно к третьей и четвертой строкам. Получим матрицу Прямой ход метода Гаусса закончен. У полученной матрицы легко определить ранг, ее базисный минор . Отсюда следует, что . По  теореме 15.3 число решений в фундаментальной системе равно разности между числом неизвестных и рангом матрицы, в нашем случае фундаментальная система состоит из трех решений.

Переходим к системе уравнений

Неизвестные и оставляем в левой части, остальные переносим в правую часть:

Положим , . Получим , . Первое решение из фундаментальной системы: .

Положим , . Получим , . Второе решение из фундаментальной системы решений: .

Положим , . Получим , . Третье решение из фундаментальной системы решений: . Фундаментальная система решений найдена. Общее решение имеет вид

Ответ: Фундаментальная система решений:
, , , общее решение: .         

        Замечание 15.7   Если решения, составляющие фундаментальную систему, умножить на любые ненулевые числа, то вновь полученные решения снова будут образовывать фундаментальную систему. Поэтому в предыдущем примере фундаментальную систему образуют и такие решения:
, , . Общее решение можно записать так: .         

Математика, вышка, высшая математика, математика онлайн, вышка онлайн, онлайн математика, онлайн решение математики, ход решения, процес решения, решение, задачи, задачи по математике, математические задачи, решение математики онлайн, решение математики online, online решение математики, решение высшей математики, решение высшей математики онлайн, матрицы, решение матриц онлайн, векторная алгебра онлайн, решение векторов онлайн, система линейных уравнений, метод Крамера, метод Гаусса, метод обратной матрицы, уравнения, системы уравнений, производные, пределы, интегралы, функция, неопределенный интеграл, определенный интеграл, решение интегралов, вычисление интегралов, решение производных, интегралы онлайн, производные онлайн, пределы онлайн, предел функции, предел последовательности, высшие производные, производная неявной функции

Решение систем неравенств методом гаусса онлайн. Метод Гаусса для решения матриц. Решение системы линейных уравнений методом Гаусса

Определение и описание метода Гаусса

Метод преобразований Гаусса (также известный как преобразование методом последовательного исключения неизвестных переменных из уравнения или матрицы) для решения систем линейных уравнений представляет собой классический методом решения системы алгебраических уравнений (СЛАУ). Также этот классический метод используют для решения таких задач как получение обратных матриц и определения ранговости матрицы.

Преобразование с помощью метода Гаусса заключается в совершении небольших (элементарных) последовательных изменениях системы линейных алгебраических уравнений, приводящих к исключению переменных из неё сверху вниз с образованием новой треугольной системы уравнений, являющейся равносильной исходной.

Определение 1

Эта часть решения носит название прямого хода решения Гаусса, так как весь процесс осуществляется сверху вниз.

После приведения исходной системы уравнений к треугольной осуществляется нахождение всех переменных системы снизу вверх (то есть первые найденные переменные занимают находятся именно на последних строчках системы или матрицы). Эта часть решения известна также как обратный ход решения методом Гаусса. Заключается его алгоритм в следующем: сначала вычисляется переменные, находящиеся ближе всего к низу системы уравнений или матрицы, затем полученные значения подставляются выше и таким образом находится ещё одна переменная и так далее.

Описание алгоритма метода Гаусса

Последовательность действий для общего решения системы уравнения методом Гаусса заключается в поочередном применении прямого и обратного хода к матрице на основе СЛАУ. Пусть исходная система уравнений имеет следующий вид:

$\begin{cases} a_{11} \cdot x_1 +…+ a_{1n} \cdot x_n = b_1 \\ … \\ a_{m1} \cdot x_1 + a_{mn} \cdot x_n = b_m \end{cases}$

Чтобы решить СЛАУ методом Гаусса, необходимо записать исходную систему уравнений в виде матрицы:

$A = \begin{pmatrix} a_{11} & … & a_{1n} \\ \vdots & … & \vdots \\ a_{m1} & … & a_{mn} \end{pmatrix}$, $b=\begin{pmatrix} b_1 \\ \vdots \\ b_m \end{pmatrix}$

Матрица $A$ называется основной матрицей и представляет собой записанные по порядку коэффициенты при переменных, а $b$ называется столбцом её свободных членов. Матрица $A$, записанная через черту со столбцом свободных членов называется расширенной матрицей:

$A = \begin{array}{ccc|c} a_{11} & … & a_{1n} & b_1 \\ \vdots & … & \vdots & …\\ a_{m1} & … & a_{mn} & b_m \end{array}$

Теперь необходимо с помощью элементарных преобразований над системой уравнений (или над матрицей, так как это удобнее) привести её к следующему виду:

$\begin{cases} α_{1j_{1}} \cdot x_{j_{1}} + α_{1j_{2}} \cdot x_{j_{2}}…+ α_{1j_{r}} \cdot x_{j_{r}} +… α_{1j_{n}} \cdot x_{j_{n}} = β_1 \\ α_{2j_{2}} \cdot x_{j_{2}}…+ α_{2j_{r}} \cdot x_{j_{r}} +… α_{2j_{n}} \cdot x_{j_{n}} = β_2 \\ …\\ α_{rj_{r}} \cdot x_{j_{r}} +… α_{rj_{n}} \cdot x_{j_{n}} = β_r \\ 0 = β_(r+1) \\ … \\ 0 = β_m \end{cases}$ (1)

Матрица, полученная из коэффициентов преобразованной системы уравнения (1) называется ступенчатой, вот так обычно выглядят ступенчатые матрицы:

$A = \begin{array}{ccc|c} a_{11} & a_{12} & a_{13} & b_1 \\ 0 & a_{22} & a_{23} & b_2\\ 0 & 0 & a_{33} & b_3 \end{array}$

Для этих матриц характерен следующий набор свойств:

  1. Все её нулевые строки стоят после ненулевых
  2. Если некоторая строка матрицы с номером $k$ ненулевая, то в предыдущей строчке этой же матрицы нулей меньше, чем в этой, обладающей номером $k$.

После получения ступенчатой матрицы необходимо подставить полученные переменные в оставшиеся уравнения (начиная с конца) и получить оставшиеся значения переменных.

Основные правила и разрешаемые преобразования при использовании метода Гаусса

При упрощении матрицы или системы уравнений этим методом нужно использовать только элементарные преобразования.

Таким преобразованиями считаются операции, которые возможно применять к матрице или системе уравнений без изменения её смысла:

  • перестановка нескольких строк местами,
  • прибавление или вычитание из одной строчки матрицы другой строчки из неё же,
  • умножение или деление строчки на константу, не равную нулю,
  • строчку, состоящую из одних нулей, полученную в процессе вычисления и упрощения системы, нужно удалить,
  • Также нужно удалить лишние пропорциональные строчки, выбрав для системы единственную из них с более подходящими и удобными для дальнейших вычислений коэффициентами.

Все элементарные преобразования являются обратимыми.

Разбор трёх основных случаев, возникающих при решении линейных уравнений используя метод простых преобразований Гаусса

Различают три возникающих случая при использовании метода Гаусса для решения систем:

  1. Когда система несовместная, то есть у неё нет каких-либо решений
  2. У системы уравнений есть решение, причём единственное, а количество ненулевых строк и столбцов в матрице равно между собой.
  3. Система имеет некое количество или множество возможных решений, а количество строк в ней меньше чем количество столбцов.

Исход решения с несовместной системой

Для этого варианта при решении матричного уравнения методом Гаусса характерно получение какой-то строчки с невозможностью выполнения равенства. Поэтому при возникновении хотя бы одного неправильного равенства полученная и исходная системы не имеют решений вне зависимости от остальных уравнений, которые они содержат. Пример несовместной матрицы:

$\begin{array}{ccc|c} 2 & -1 & 3 & 0 \\ 1 & 0 & 2 & 0\\ 0 & 0 & 0 & 1 \end{array}$

В последней строчке возникло невыполняемое равенство: $0 \cdot x_{31} + 0 \cdot x_{32} + 0 \cdot x_{33} = 1$.

Система уравнений, у которой есть только одно решение

Данные системы после приведения к ступенчатой матрице и удаления строчек с нулями имеют одинаковое количество строк и столбцов в основной матрице. Вот простейший пример такой системы:

$\begin{cases} x_1 — x_2 = -5 \\ 2 \cdot x_1 + x_2 = -7 \end{cases}$

Запишем её в виде матрицы:

$\begin{array}{cc|c} 1 & -1 & -5 \\ 2 & 1 & -7 \end{array}$

Чтобы привести первую ячейку второй строчки к нулю, домножим верхнюю строку на $-2$ и вычтем её из нижней строчки матрицы, а верхнюю строчку оставим в исходном виде, в итоге имеем следующее:

$\begin{array}{cc|c} 1 & -1 & -5 \\ 0 & 3 & 10 \end{array}$

Этот пример можно записать в виде системы:

$\begin{cases} x_1 — x_2 = -5 \\ 3 \cdot x_2 = 10 \end{cases}$

Из нижнего уравнения выходит следующее значение $x$: $x_2 = 3 \frac{1}{3}$. Подставим это значение в верхнее уравнение: $x_1 – 3 \frac{1}{3}$, получаем $x_1 = 1 \frac{2}{3}$.

Система, обладающая множеством возможных вариантов решений

Для этой системы характерно меньшее количество значащих строк, чем количество столбцов в ней (учитываются строки основной матрицы).

Переменные в такой системе делятся на два вида: базисные и свободные. При преобразовании такой системы содержащиеся в ней основные переменные необходимо оставить в левой области до знака “=”, а остальные переменные перенести в правую часть равенства.

У такой системы есть только некое общее решение.

Разберём следующую систему уравнений:

$\begin{cases} 2y_1 + 3y_2 + x_4 = 1 \\ 5y_3 — 4y_4 = 1 \end{cases}$

Запишем её в виде матрицы:

$\begin{array}{cccc|c} 2 & 3 & 0 & 1 & 1 \\ 0 & 0 & 5 & 4 & 1 \\ \end{array}$

Наша задача найти общее решение системы. Для этой матрицы базисными переменными будут $y_1$ и $y_3$ (для $y_1$ — так как он стоит на первом месте, а в случае $y_3$ — располагается после нулей).

В качестве базисных переменных выбираем именно те, которые первые в строке не равны нулю.

Оставшиеся переменные называются свободными, через них нам необходимо выразить базисные.

Используя так называемый обратный ход, разбираем систему снизу вверх, для этого сначала выражаем $y_3$ из нижней строчки системы:

$5y_3 – 4y_4 = 1$

$5y_3 = 4y_4 + 1$

$y_3 = \frac{4/5}y_4 + \frac{1}{5}$.

Теперь в верхнее уравнение системы $2y_1 + 3y_2 + y_4 = 1$ подставляем выраженное $y_3$: $2y_1 + 3y_2 — (\frac{4}{5}y_4 + \frac{1}{5}) + y_4 = 1$

Выражаем $y_1$ через свободные переменные $y_2$ и $y_4$:

$2y_1 + 3y_2 — \frac{4}{5}y_4 — \frac{1}{5} + y_4 = 1$

$2y_1 = 1 – 3y_2 + \frac{4}{5}y_4 + \frac{1}{5} – y_4$

$2y_1 = -3y_2 — \frac{1}{5}y_4 + \frac{6}{5}$

$y_1 = -1.5x_2 – 0.1y_4 + 0.6$

Решение готово.

Пример 1

Решить слау методом Гаусса. Примеры. Пример решения системы линейных уравнений заданных матрицей 3 на 3 используя метод Гаусса

$\begin{cases} 4x_1 + 2x_2 – x_3 = 1 \\ 5x_1 + 3x_2 — 2x^3 = 2\\ 3x_1 + 2x_2 – 3x_3 = 0 \end{cases}$

Запишем нашу систему в виде расширенной матрицы:

$\begin{array}{ccc|c} 4 & 2 & -1 & 1 \\ 5 & 3 & -2 & 2 \\ 3 & 2 & -3 & 0\\ \end{array}$

Теперь для удобства и практичности нужно преобразовать матрицу так, чтобы в верхнем углу крайнего столбца была $1$.

Для этого к 1-ой строчке нужно прибавляем строчку из середины, умноженную на $-1$, а саму среднюю строчку записываем как есть, выходит:

$\begin{array}{ccc|c} -1 & -1 & 1 & -1 \\ 5 & 3 & -2 & 2 \\ 3 & 2 & -3 & 0\\ \end{array}$

$\begin{array}{ccc|c} -1 & -1 & 1 & -1 \\ 0 & -2 & 3 & -3 \\ 0 & -1 & 0 & -3\\ \end{array}$

Домножим верхнюю и последнюю строчки на $-1$, а также поменяем местами последнюю и среднюю строки:

$\begin{array}{ccc|c} 1 & 1 & -1 & 1 \\ 0 & 1 & 0 & 3 \\ 0 & -2 & 3 & -3\\ \end{array}$

$\begin{array}{ccc|c} 1 & 1 & -1 & 1 \\ 0 & 1 & 0 & 3 \\ 0 & 0 & 3 & 3\\ \end{array}$

И разделим последнюю строчку на $3$:

$\begin{array}{ccc|c} 1 & 1 & -1 & 1 \\ 0 & 1 & 0 & 3 \\ 0 & 0 & 1 & 1\\ \end{array}$

Получаем следующую систему уравнений, равносильную исходной:

$\begin{cases} x_1 + x_2 – x_3 = 1\\ x_2 = 3 \\ x_3 = 1 \end{cases}$

Из верхнего уравнения выражаем $x_1$:

$x1 = 1 + x_3 – x_2 = 1 + 1 – 3 = -1$.

Пример 2

Пример решения системы, заданной с помощью матрицы 4 на 4 методом Гаусса

$\begin{array}{cccc|c} 2 & 5 & 4 & 1 & 20 \\ 1 & 3 & 2 & 1 & 11 \\ 2 & 10 & 9 & 7 & 40\\ 3 & 8 & 9 & 2 & 37 \\ \end{array}$.

В начале меняем местами верхнюю исследующую за ней строчки, чтобы получить в левом верхнем углу $1$:

$\begin{array}{cccc|c} 1 & 3 & 2 & 1 & 11 \\ 2 & 5 & 4 & 1 & 20 \\ 2 & 10 & 9 & 7 & 40\\ 3 & 8 & 9 & 2 & 37 \\ \end{array}$.

Теперь умножим верхнюю строчку на $-2$ и прибавим ко 2-ой и к 3-ьей. К 4-ой прибавляем 1-ую строку, домноженную на $-3$:

$\begin{array}{cccc|c} 1 & 3 & 2 & 1 & 11 \\ 0 & -1 & 0 & -1 & -2 \\ 0 & 4 & 5 & 5 & 18\\ 0 & -1 & 3 & -1 & 4 \\ \end{array}$

Теперь к строке с номером 3 прибавляем строку 2, умноженную на $4$, а к строке 4 прибавляем строку 2, умноженную на $-1$.

$\begin{array}{cccc|c} 1 & 3 & 2 & 1 & 11 \\ 0 & -1 & 0 & -1 & -2 \\ 0 & 0 & 5 & 1 & 10\\ 0 & 0 & 3 & 0 & 6 \\ \end{array}$

Домножаем строку 2 на $-1$, а строку 4 делим на $3$ и ставим на место строки 3.

$\begin{array}{cccc|c} 1 & 3 & 2 & 1 & 11 \\ 0 & 1 & 0 & 1 & 2 \\ 0 & 0 & 1 & 0 & 2\\ 0 & 0 & 5 & 1 & 10 \\ \end{array}$

Теперь прибавляем к последней строке предпоследнюю, домноженную на $-5$.

$\begin{array}{cccc|c} 1 & 3 & 2 & 1 & 11 \\ 0 & 1 & 0 & 1 & 2 \\ 0 & 0 & 1 & 0 & 2\\ 0 & 0 & 0 & 1 & 0 \\ \end{array}$

Решаем полученную систему уравнений:

$\begin{cases} m = 0 \\ g = 2\\ y + m = 2\ \ x + 3y + 2g + m = 11\end{cases}$

Еще с начала XVI-XVIII веков математики усиленно начали изучать функции, благодаря которым так много в нашей жизни изменилось. Компьютерная техника без этих знаний просто не существовала бы. Для решения сложных задач, линейных уравнений и функций были созданы различные концепции, теоремы и методики решения. Одним из таких универсальных и рациональных способов и методик решения линейных уравнений и их систем стал и метод Гаусса. Матрицы, их ранг, детерминант — все можно посчитать, не используя сложных операций.

Что представляет собой СЛАУ

В математике существует понятие СЛАУ — система линейных алгебраических уравнений. Что же она собой представляет? Это набор из m уравнений с искомыми n неизвестными величинами, обычно обозначающимися как x, y, z, или x 1 , x 2 … x n, или другими символами. Решить методом Гаусса данную систему — означает найти все искомые неизвестные. Если система имеет одинаковое число неизвестных и уравнений, тогда она называется системой n-го порядка.

Наиболее популярные методы решения СЛАУ

В учебных заведениях среднего образования изучают различные методики решения таких систем. Чаще всего это простые уравнения, состоящие из двух неизвестных, поэтому любой существующий метод для поиска ответа на них не займет много времени. Это может быть как метод подстановки, когда из одного уравнения выводится другое и подставляется в изначальное. Или метод почленного вычитания и сложения. Но наиболее легким и универсальным считается метод Гаусса. Он дает возможность решать уравнения с любым количеством неизвестных. Почему именно эта методика считается рациональной? Все просто. Матричный способ хорош тем, что здесь не требуется по несколько раз переписывать ненужные символы в виде неизвестных, достаточно проделать арифметические операции над коэффициентами — и получится достоверный результат.

Где используются СЛАУ на практике

Решением СЛАУ являются точки пересечения прямых на графиках функций. В наш высокотехнологический компьютерный век людям, которые тесно связаны с разработкой игр и прочих программ, необходимо знать, как решать такие системы, что они представляют и как проверить правильность получившегося результата. Наиболее часто программисты разрабатывают специальные программы-вычислители линейной алгебры, сюда входит и система линейных уравнений. Метод Гаусса позволяет высчитать все существующие решения. Также используются и другие упрощенные формулы и методики.

Критерий совместимости СЛАУ

Такую систему можно решить только в том случае, если она совместима. Для понятности представим СЛАУ в виде Ax=b. Она имеет решение, если rang(A) равняется rang(A,b). В этом случае (A,b) — это матрица расширенного вида, которую можно получить из матрицы А, переписав ее со свободными членами. Выходит, что решить линейные уравнения методом Гаусса достаточно легко.

Возможно, некоторые обозначения не совсем понятны, поэтому необходимо рассмотреть все на примере. Допустим, есть система: x+y=1; 2x-3y=6. Она состоит всего из двух уравнений, в которых 2 неизвестные. Система будет иметь решение только в том случае, если ранг ее матрицы будет равняться рангу расширенной матрицы. Что такое ранг? Это число независимых строк системы. В нашем случае ранг матрицы 2. Матрица А будет состоять из коэффициентов, находящихся возле неизвестных, а в расширенную матрицу вписываются и коэффициенты, находящиеся за знаком «=».

Почему СЛАУ можно представить в матричном виде

Исходя из критерия совместимости по доказанной теореме Кронекера-Капелли, систему линейных алгебраических уравнений можно представить в матричном виде. Применяя каскадный метод Гаусса, можно решить матрицу и получить единственный достоверный ответ на всю систему. Если ранг обычной матрицы равняется рангу ее расширенной матрицы, но при этом меньше количества неизвестных, тогда система имеет бесконечное количество ответов.

Преобразования матриц

Прежде чем переходить к решению матриц, необходимо знать, какие действия можно проводить над их элементами. Существует несколько элементарных преобразований:

  • Переписывая систему в матричный вид и осуществляя ее решение, можно умножать все элементы ряда на один и тот же коэффициент.
  • Для того чтобы преобразовать матрицу в канонический вид, можно менять местами два параллельных ряда. Канонический вид подразумевает, что все элементы матрицы, которые расположены по главной диагонали, становятся единицами, а оставшиеся — нулями.
  • Соответствующие элементы параллельных рядов матрицы можно прибавлять один к другому.

Метод Жордана-Гаусса

Суть решения систем линейных однородных и неоднородных уравнений методом Гаусса в том, чтобы постепенно исключить неизвестные. Допустим, у нас есть система из двух уравнений, в которых две неизвестные. Чтобы их найти, необходимо проверить систему на совместимость. Уравнение методом Гаусса решается очень просто. Необходимо выписать коэффициенты, находящиеся возле каждого неизвестного в матричный вид. Для решения системы понадобится выписать расширенную матрицу. Если одно из уравнений содержит меньшее количество неизвестных, тогда на место пропущенного элемента необходимо поставить «0». К матрице применяются все известные методы преобразования: умножение, деление на число, прибавление соответствующих элементов рядов друг к другу и другие. Получается, что в каждом ряду необходимо оставить одну переменную со значением «1», остальные привести к нулевому виду. Для более точного понимания необходимо рассмотреть метод Гаусса на примерах.

Простой пример решения системы 2х2

Для начала возьмем простенькую систему алгебраических уравнений, в которой будет 2 неизвестных.

Перепишем ее в расширенную матрицу.

Чтобы решить данную систему линейных уравнений, требуется проделать всего две операции. Нам необходимо привести матрицу к каноническому виду, чтобы по главной диагонали стояли единицы. Так, переводя с матричного вида обратно в систему, мы получим уравнения: 1x+0y=b1 и 0x+1y=b2, где b1 и b2 — получившиеся ответы в процессе решения.

  1. Первое действие при решении расширенной матрицы будет таким: первый ряд необходимо умножить на -7 и прибавить соответственно отвечающие элементы ко второй строке, чтобы избавиться от одного неизвестного во втором уравнении.
  2. Так как решение уравнений методом Гаусса подразумевает приведение матрицы к каноническому виду, тогда необходимо и с первым уравнением проделать те же операции и убрать вторую переменную. Для этого вторую строку отнимаем от первой и получаем необходимый ответ — решение СЛАУ. Или, как показано на рисунке, вторую строку умножаем на коэффициент -1 и прибавляем к первой строке элементы второго ряда. Это одно и то же.

Как видим, наша система решена методом Жордана-Гаусса. Переписываем ее в необходимую форму: x=-5, y=7.

Пример решения СЛАУ 3х3

Предположим, что у нас есть более сложная система линейных уравнений. Метод Гаусса дает возможность высчитать ответ даже для самой, казалось бы, запутанной системы. Поэтому, чтобы более глубоко вникнуть в методику расчета, можно переходить к более сложному примеру с тремя неизвестными.

Как и в прежнем примере, переписываем систему в вид расширенной матрицы и начинаем приводить ее к каноническому виду.

Для решения этой системы понадобится произвести гораздо больше действий, чем в предыдущем примере.

  1. Сначала необходимо сделать в первом столбце один единичный элемент и остальные нули. Для этого умножаем первое уравнение на -1 и прибавляем к нему второе уравнение. Важно запомнить, что первую строку мы переписываем в изначальном виде, а вторую — уже в измененном.
  2. Далее убираем эту же первую неизвестную из третьего уравнения. Для этого элементы первой строки умножаем на -2 и прибавляем их к третьему ряду. Теперь первая и вторая строки переписываются в изначальном виде, а третья — уже с изменениями. Как видно по результату, мы получили первую единицу в начале главной диагонали матрицы и остальные нули. Еще несколько действий, и система уравнений методом Гаусса будет достоверно решена.
  3. Теперь необходимо проделать операции и над другими элементами рядов. Третье и четвертое действие можно объединить в одно. Нужно разделить вторую и третью строку на -1, чтобы избавиться от минусовых единиц по диагонали. Третью строку мы уже привели к необходимому виду.
  4. Дальше приведем к каноническому виду вторую строку. Для этого элементы третьего ряда умножаем на -3 и прибавляем их ко второй строчке матрицы. Из результата видно, что вторая строка тоже приведена к необходимой нам форме. Осталось проделать еще несколько операций и убрать коэффициенты неизвестных из первой строки.
  5. Чтобы из второго элемента строки сделать 0, необходимо умножить третью строку на -3 и прибавить ее к первому ряду.
  6. Следующим решающим этапом будет прибавление к первой строке необходимые элементы второго ряда. Так мы получаем канонический вид матрицы, а, соответственно, и ответ.

Как видно, решение уравнений методом Гаусса довольно простое.

Пример решения системы уравнений 4х4

Некоторые более сложные системы уравнений можно решить методом Гаусса посредством компьютерных программ. Необходимо вбить в существующие пустые ячейки коэффициенты при неизвестных, и программа сама пошагово рассчитает необходимый результат, подробно описывая каждое действие.

Ниже описана пошаговая инструкция решения такого примера.

В первом действии в пустые ячейки вписываются свободные коэффициенты и числа при неизвестных. Таким образом, получается такая же расширенная матрица, которую мы пишем вручную.

И производятся все необходимые арифметические операции, чтобы привести расширенную матрицу к каноническому виду. Необходимо понимать, что не всегда ответ на систему уравнений — это целые числа. Иногда решение может быть из дробных чисел.

Проверка правильности решения

Метод Жордана-Гаусса предусматривает проверку правильности результата. Для того чтобы узнать, правильно ли посчитаны коэффициенты, необходимо всего-навсего подставить результат в изначальную систему уравнений. Левая сторона уравнения должна соответствовать правой стороне, находящейся за знаком «равно». Если ответы не совпадают, тогда необходимо пересчитывать заново систему или попробовать применить к ней другой известный вам метод решения СЛАУ, такой как подстановка или почленное вычитание и сложение. Ведь математика — это наука, которая имеет огромное количество различных методик решения. Но помните: результат должен быть всегда один и тот же, независимо от того, какой метод решения вы использовали.

Метод Гаусса: наиболее часто встречающиеся ошибки при решении СЛАУ

Во время решения линейных систем уравнений чаще всего возникают такие ошибки, как неправильный перенос коэффициентов в матричный вид. Бывают системы, в которых отсутствуют в одном из уравнений некоторые неизвестные, тогда, перенося данные в расширенную матрицу, их можно потерять. В результате при решении данной системы результат может не соответствовать действительному.

Еще одной из главных ошибок может быть неправильное выписывание конечного результата. Нужно четко понимать, что первый коэффициент будет соответствовать первому неизвестному из системы, второй — второму, и так далее.

Метод Гаусса подробно описывает решение линейных уравнений. Благодаря ему легко произвести необходимые операции и найти верный результат. Кроме того, это универсальное средство для поиска достоверного ответа на уравнения любой сложности. Может быть, поэтому его так часто используют при решении СЛАУ.

Здесь вы сможете бесплатно решить систему линейных уравнений методом Гаусса онлайн больших размеров в комплексных числах с очень подробным решением. Наш калькулятор умеет решать онлайн как обычную определенную, так и неопределенную систему линейных уравнений методом Гаусса, которая имеет бесконечное множество решений. В этом случае в ответе вы получите зависимость одних переменных через другие, свободные. Также можно проверить систему уравнений на совместность онлайн, используя решение методом Гаусса.

О методе

При решении системы линейных уравнений онлайн методом Гаусса выполняются следующие шаги.

  1. Записываем расширенную матрицу.
  2. Фактически решение разделяют на прямой и обратный ход метода Гаусса. Прямым ходом метода Гаусса называется приведение матрицы к ступенчатому виду. Обратным ходом метода Гаусса называется приведение матрицы к специальному ступенчатому виду. Но на практике удобнее сразу занулять то, что находится и сверху и снизу рассматриваемого элемента. Наш калькулятор использует именно этот подход.
  3. Важно отметить, что при решении методом Гаусса, наличие в матрице хотя бы одной нулевой строки с НЕнулевой правой частью (столбец свободных членов) говорит о несовместности системы. Решение линейной системы в таком случае не существует.

Чтобы лучше всего понять принцип работы алгоритма Гаусса онлайн введите любой пример, выберите «очень подробное решение» и посмотрите его решение онлайн.

1. Система линейных алгебраических уравнений

1.1 Понятие системы линейных алгебраических уравнений

Система уравнений – это условие, состоящее в одновременном выполнении нескольких уравнений относительно нескольких переменных. Системой линейных алгебраических уравнений (далее – СЛАУ), содержащей m уравнений и n неизвестных, называется система вида:

где числа a ij называются коэффициентами системы, числа b i – свободными членами, a ij и b i (i=1,…, m; b=1,…, n) представляют собой некоторые известные числа, а x 1 ,…, x n – неизвестные. В обозначении коэффициентов a ij первый индекс i обозначает номер уравнения, а второй j – номер неизвестного, при котором стоит этот коэффициент. Подлежат нахождению числа x n . Такую систему удобно записывать в компактной матричной форме: AX=B. Здесь А – матрица коэффициентов системы, называемая основной матрицей;

– вектор-столбец из неизвестных xj.
– вектор-столбец из свободных членов bi.

Произведение матриц А*Х определено, так как в матрице А столбцов столько же, сколько строк в матрице Х (n штук).

Расширенной матрицей системы называется матрица A системы, дополненная столбцом свободных членов

1.2 Решение системы линейных алгебраических уравнений

Решением системы уравнений называется упорядоченный набор чисел (значений переменных), при подстановке которых вместо переменных каждое из уравнений системы обращается в верное равенство.

Решением системы называется n значений неизвестных х1=c1, x2=c2,…, xn=cn, при подстановке которых все уравнения системы обращаются в верные равенства. Всякое решение системы можно записать в виде матрицы-столбца

Система уравнений называется совместной, если она имеет хотя бы одно решение, и несовместной, если она не имеет ни одного решения.

Совместная система называется определенной, если она имеет единственное решение, и неопределенной, если она имеет более одного решения. В последнем случае каждое ее решение называется частным решением системы. Совокупность всех частных решений называется общим решением.

Решить систему – это значит выяснить, совместна она или несовместна. Если система совместна, найти ее общее решение.

Две системы называются эквивалентными (равносильными), если они имеют одно и то же общее решение. Другими словами, системы эквивалентны, если каждое решение одной из них является решением другой, и наоборот.

Преобразование, применение которого превращает систему в новую систему, эквивалентную исходной, называется эквивалентным или равносильным преобразованием. Примерами эквивалентных преобразований могут служить следующие преобразования: перестановка местами двух уравнений системы, перестановка местами двух неизвестных вместе с коэффициентами у всех уравнений, умножение обеих частей какого-либо уравнения системы на отличное от нуля число.

Система линейных уравнений называется однородной, если все свободные члены равны нулю:

Однородная система всегда совместна, так как x1=x2=x3=…=xn=0 является решением системы. Это решение называется нулевым или тривиальным.

2. Метод исключения Гаусса

2.1 Сущность метода исключения Гаусса

Классическим методом решения систем линейных алгебраических уравнений является метод последовательного исключения неизвестных – метод Гаусса (его еще называют методом гауссовых исключений). Это метод последовательного исключения переменных, когда с помощью элементарных преобразований система уравнений приводится к равносильной системе ступенчатого (или треугольного) вида, из которого последовательно, начиная с последних (по номеру) переменных, находятся все остальные переменные.

Процесс решения по методу Гаусса состоит из двух этапов: прямой и обратный ходы.

1. Прямой ход.

На первом этапе осуществляется так называемый прямой ход, когда путём элементарных преобразований над строками систему приводят к ступенчатой или треугольной форме, либо устанавливают, что система несовместна. А именно, среди элементов первого столбца матрицы выбирают ненулевой, перемещают его на крайнее верхнее положение перестановкой строк и вычитают получившуюся после перестановки первую строку из остальных строк, домножив её на величину, равную отношению первого элемента каждой из этих строк к первому элементу первой строки, обнуляя тем самым столбец под ним.

После того, как указанные преобразования были совершены, первую строку и первый столбец мысленно вычёркивают и продолжают пока не останется матрица нулевого размера. Если на какой-то из итераций среди элементов первого столбца не нашёлся ненулевой, то переходят к следующему столбцу и проделывают аналогичную операцию.

На первом этапе (прямой ход) система приводится к ступенчатому (в частности, треугольному) виду.

Приведенная ниже система имеет ступенчатый вид:

,

Коэффициенты aii называются главными (ведущими) элементами системы.

(если a11=0, переставим строки матрицы так, чтобы a 11 не был равен 0. Это всегда возможно, т. к. в противном случае матрица содержит нулевой столбец, ее определитель равен нулю и система несовместна).

Преобразуем систему, исключив неизвестное х1 во всех уравнениях, кроме первого (используя элементарные преобразования системы). Для этого умножим обе части первого уравнения на

и сложим почленно со вторым уравнением системы (или из второго уравнения почленно вычтем первое, умноженное на ). Затем умножим обе части первого уравнения на и сложим с третьим уравнением системы (или из третьего почленно вычтем первое, помноженное на ). Таким образом, последовательно умножаем первую строку на число и прибавляем к i -й строке, для i= 2, 3, …, n.

Продолжая этот процесс, получим эквивалентную систему:


– новые значения коэффициентов при неизвестных и свободные члены в последних m-1 уравнениях системы, которые определяются формулами:

Таким образом, на первом шаге уничтожаются все коэффициенты, лежащие под первым ведущим элементом a 11

0, на втором шаге уничтожаются элементы, лежащие под вторым ведущим элементом а 22 (1) (если a 22 (1) 0) и т.д. Продолжая этот процесс и дальше, мы, наконец, на (m-1) шаге приведем исходную систему к треугольной системе.

Если в процессе приведения системы к ступенчатому виду появятся нулевые уравнения, т.е. равенства вида 0=0, их отбрасывают. Если же появится уравнение вида

то это свидетельствует о несовместности системы.

На этом прямой ход метода Гаусса заканчивается.

2. Обратный ход.

На втором этапе осуществляется так называемый обратный ход, суть которого заключается в том, чтобы выразить все получившиеся базисные переменные через небазисные и построить фундаментальную систему решений, либо, если все переменные являются базисными, то выразить в численном виде единственное решение системы линейных уравнений.

Эта процедура начинается с последнего уравнения, из которого выражают соответствующую базисную переменную (она в нем всего одна) и подставляют в предыдущие уравнения, и так далее, поднимаясь по «ступенькам» наверх.

Каждой строчке соответствует ровно одна базисная переменная, поэтому на каждом шаге, кроме последнего (самого верхнего), ситуация в точности повторяет случай последней строки.

Примечание: на практике удобнее работать не с системой, а с расширенной ее матрицей, выполняя все элементарные преобразования над ее строками. Удобно, чтобы коэффициент a11 был равен 1 (уравнения переставить местами, либо разделить обе части уравнения на a11).

2.2 Примеры решения СЛАУ методом Гаусса

В данном разделе на трех различных примерах покажем, как методом Гаусса можно решить СЛАУ.

Пример 1. Решить СЛАУ 3-го порядка.

Обнулим коэффициенты при

во второй и третьей строчках. Для этого домножим их на 2/3 и 1 соответственно и сложим с первой строкой:

Одним из универсальных и эффективных методов реше­ния линейных алгебраических систем является метод Гаусса , состо­ящий в последовательном исключении неизвестных.

Напомним, две системы называются эквивалентными (равносильными), если множества их решений совпадают. Другими словами, системы эквивалентны, если каждое решение одной из них является решением другой и наоборот. Эквивалентные системы получаются приэлементарных преобразованиях уравнений системы:

    умножение обеих частей уравнения на число отличное от нуля;

    прибавление к некоторому уравнению соответствующих частей другого уравнения, умноженных на число отличное от нуля;

    перестановка двух уравнений.

Пусть дана система уравнений

Процесс решения этой системы по методу Гаусса состоит из двух этапов. На первом этапе (прямой ход) система с помощью элементарных преобразований приводится к ступен­чатому , илитреугольному виду, а на втором этапе (обратный ход) идет последовательное, начиная с последнего по номеру переменного, определение неизвестных из полученной ступенчатой системы.

Предположим, что коэффициент данной системы
, в против­ном случае в системе первую строку можно поменять местами с любой другой строкой так, чтобы коэффициент прибыл отличен от нуля.

Преобразуем систему, исключив неизвестное во всех уравне­ниях, кроме первого. Для этого умножим обе части первого уравнения наи сложим почленно со вторым уравнением системы. Затем умножим обе части первого уравнения наи сложим с третьим уравнением системы. Продолжая этот процесс, получим эквивалент­ную систему

Здесь
– новые значения коэффициентов и свободных членов, которые получаются после первого шага.

Аналогичным образом, считая главным элементом
, исклю­чим неизвестноеиз всех уравнений системы, кроме первого и второго. Продолжим этот процесс, пока это возможно, в результате получим ступенчатую систему

,

где ,
,…,– главные элементы системы
.

Если в процессе приведения системы к ступенчатому виду появятся уравнения , т. е. равенства вида
, их отбрасывают, так как им удовлетворяют любые наборы чисел
. Если же при
появится уравнение вида, которое не имеет решений, то это свидетельствует о несовместности системы.

При обратном ходе из последнего уравнения преобразованной сту­пенчатой системы выражается первое неизвестное через все остальные неизвестные
, которые называютсвободными . Затем выражение переменнойиз последнего уравнения системы подставляется в предпоследнее уравнение и из него выражается переменная
. Аналогичным образом последовательно определяются переменные
. Переменные
, выраженные через свободные переменные, называютсябазисными (зависимыми). В результате получается общее решение системы линейных уравнений.

Чтобы найти частное решение системы, свободным неизвестным
в общем решении придаются произвольные значения и вычисляются значения переменных
.

Технически удобнее подвергать элементарным преобразованиям не сами уравнения системы, а расширенную матрицу системы

.

Метод Гаусса — универсальный метод, который позволяет решать не только квадратные, но и прямоугольные системы, в которых число неизвестных
не равно числу уравнений
.

Достоинство этого метода состоит также в том, что в процессе решения мы одновременно исследуем систему на совместность, так как, приведя расширенную матрицу
к ступенчатому виду, легко определить ранги матрицыи расширенной матрицы
и применитьтеорему Кронекера — Капелли .

Пример 2.1 Методом Гаусса решить систему

Решение . Число уравнений
и число неизвестных
.

Составим расширенную матрицу системы, приписав справа от матрицы коэффициентов столбец свободных членов.

Приведём матрицу к треугольному виду; для этого будем получать «0» ниже элементов, стоящих на главной диагонали с помощью элементарных преобразований.

Чтобы получить «0» во второй позиции первого столбца, умножим первую строку на (-1) и прибавим ко второй строке.

Это преобразование запишем числом (-1) против первой строки и обозначим стрелкой, идущей от первой строки ко второй строке.

Для получения «0» в третьей позиции первого столбца, умножим первую строку на (-3) и прибавим к третьей строке; покажем это действие с помощью стрелки, идущей от первой строки к третьей.




.

В полученной матрице, записанной второй в цепочке матриц, получим «0» во втором столбце в третьей позиции. Для этого умножили вторую строку на (-4) и прибавили к третьей. В полученной матрице вторую строку умножим на (-1), а третью — разделим на (-8). Все элементы этой матрицы, лежащие ниже диагональных элементов — нули.

Так как , система является совместной и определенной.

Соответствующая последней матрице система уравнений имеет треугольный вид:

Из последнего (третьего) уравнения
. Подставим во второе уравнение и получим
.

Подставим
и
в первое уравнение, найдём


.

Решение системы линейных уравнений методом Гаусса в MS Excel

На днях понадобилось найти корни системы линейных уравнений методом Гаусса в Microsoft Excel. Готовый алгоритм решения можно найти в книге Гарнаева «Использование Excel и VBA в экономике и финансах», но объяснение там очень скудное и не совсем понятное. Постараюсь описать подробней для тех, кому может понадобиться этот алгоритм.

Лирическое отступление: в тексте будет предлагаться ввести в диапазон ячеек формулу вида: {=A1:B3+$C$2:$C$3} и т.п., это так-называемые «формулы массива» (формула, выполняющая несколько вычислений над одним или несколькими наборами значений, а затем возвращающая один или несколько результатов. Формулы массива заключены в фигурные скобки { }). Microsoft Excel автоматически заключает ее в фигурные скобки ( { } ). Для введения такого типа формул необходимо выделить весь диапазон, куда нужно вставить формулу, в первой ячейке ввести формулу без фигурных скобок (для примера выше — =A1:B3+$C$2:$C$3) и нажать Ctrl+Shift+Enter.

Пускай имеем систему линейных уравнений:

1. Запишем коэффициенты системы уравнений в ячейки A1:D4 а столбец свободных членов в ячейки E1:E4. Если в ячейке A1 находится 0, необходимо поменять строки местами так, чтоб в этой ячейке было отличное от ноля значение. Для большей наглядности можно добавить заливку ячеек, в которых находятся свободные члены.

2. Необходимо коэффициент при x1 во всех уравнениях кроме первого привести к 0. Для начала сделаем это для второго уравнения. Скопируем первую строку в ячейки A6:E6 без изменений, в ячейки A7:E7 необходимо ввести формулу: {=A2:E2-$A$1:$E$1*(A2/$A$1)}. Таким образом мы от второй строки отнимаем первую, умноженную на A2/$A$1, т.е. отношение первых коэффициентов второго и первого уравнения. Для удобства заполнения строк 8 и 9 ссылки на ячейки первой строки необходимо использовать абсолютные (используем символ $).

3. Копируем введенную формулу формулу в строки 8 и 9, таким образом избавляемся от коэффициентов перед x1 во всех уравнениях кроме первого.

4. Теперь приведем коэффициенты перед x2 в третьем и четвертом уравнении к 0. Для этого скопируем полученные 6-ю и 7-ю строки (только значения) в строки 11 и 12, а в ячейки A13:E13 введем формулу {=A8:E8-$A$7:$E$7*(B8/$B$7)}, которую затем скопируем в ячейки A14:E14. Таким образом реализуется разность строк 8 и 7, умноженных на коэффициент B8/$B$7. Не забываем проводить перестановку строк, чтоб избавиться от 0 в знаменателе дроби.

5. Осталось привести коэффициент при x3 в четвертом уравнении к 0, для этого вновь проделаем аналогичные действия: скопируем полученные 11, 12 и 13-ю строки (только значения) в строки 16-18, а в ячейки A19:E19 введем формулу {=A14:E14-$A$13:$E$13*(C14/$C$13)}. Таким образом реализуется разность строк 14 и 13, умноженных на коэффициент C14/$C$13. Не забываем проводить перестановку строк, чтоб избавиться от 0 в знаменателе дроби.

6. Прямая прогонка методом Гаусса завершена. Обратную прогонку начнем с последней строки полученной матрицы. Необходимо все элементы последней строки разделить на коэффициент при x4. Для этого в строку 24 введем формулу {=A19:E19/D19}.

7. Приведем все строки к подобному виду, для этого заполним строки 23, 22, 21 следующими формулами:
23: {=(A18:E18-A24:E24*D18)/C18} — отнимаем от третьей строки четвертую умноженную на коэффициент при x4 третьей строки.
22: {=(A17:E17-A23:E23*C17-A24:E24*D17)/B17} — от второй строки отнимаем третью и четвертую, умноженные на соответствующие коэффициенты.
21: {=(A16:E16-A22:E22*B16-A23:E23*C16-A24:E24*D16)/A16} — от первой строки отнимаем вторую, третью и четвертую, умноженные на соответствующие коэффициенты.
Результат (корни уравнения) вычислены в ячейках E21:E24.

UPDATE от 25 апреля 2012 г. Выкладываю xls-файл с решением линейных уравнений методом Гаусса в Microsoft Excel:

Форма

Row Echelon Form и сокращенная Row Echelon Form


Содержание (Щелкните, чтобы перейти к этому разделу:

  1. Что такое форма эшелона?
  2. Форма эшелона строк
  3. Форма пониженного эшелона строк
  4. Исключение по Гауссу
  5. Ранг матрицы

Посмотрите видео с определениями эшелонов, рядов и пониженных рядов:


Не можете посмотреть видео? Кликните сюда.

Что такое форма эшелона?

Форма

Echelon означает, что матрица находится в одном из двух состояний:

  • Форма рядного эшелона.
  • Уменьшенная форма рядного эшелона.

Это означает, что матрица удовлетворяет следующим трем требованиям:

  1. Первое число в строке (называемое ведущим коэффициентом) — 1. Примечание: некоторые авторы не требуют, чтобы ведущий коэффициент был равен 1; это могло быть любое число. Вы можете узнать у своего инструктора, какой версии этого правила они придерживаются).
  2. Каждая ведущая единица находится справа от первой.
  3. Любые ненулевые строки всегда находятся над строками со всеми нулями.

Следующие примеры представляют собой матрицы в эшелонированной форме:

Следующие примеры представляют собой , а не в эшелонированной форме:

Матрица A не имеет строк со всеми нулями ниже ненулевых строк.
Matrix B имеет 1 на 2 позиции в третьем ряду. Для формы эшелона строки он должен быть справа от ведущего коэффициента над ним. Другими словами, он должен быть на четвертой позиции вместо 3.
Matrix C имеет 2 в качестве ведущего коэффициента вместо 1.
Матрица D имеет -1 в качестве ведущего коэффициента вместо 1.

Другой способ думать о матрице в форме эшелона — это то, что матрица подверглась гауссовскому исключению, которое представляет собой серию операций со строками.

Уникальность и формы эшелона

Эшелонированная форма матрицы не уникальна, что означает, что при сокращении строк возможно бесконечное количество ответов. Уменьшенная форма эшелона строки находится на другом конце спектра; это уникально , что означает, что сокращение строк в матрице даст один и тот же ответ независимо от того, как вы выполняете те же операции со строками.
Вернуться к началу.

Что такое форма эшелона строк?

Матрица находится в форме эшелона строк, если она соответствует следующим требованиям:

  • Первое ненулевое число слева («ведущий коэффициент») всегда находится справа от первого ненулевого числа в строке выше.
  • Строки, состоящие из нулей, находятся внизу матрицы.

Форма рядного эшелона. «А» может представлять любое число.


Технически, старший коэффициент может быть любым числом.Однако в большинстве учебников линейной алгебры утверждается, что ведущим коэффициентом должно быть число 1. Чтобы добавить путаницы, некоторые определения формы эшелона строк утверждают, что должны быть нули как выше , так и ниже ведущего коэффициента. Поэтому лучше всего следовать определению, данному в учебнике, которому вы следуете (или тому, что дал вам ваш профессор). Если вы не уверены (то есть сейчас воскресенье, у вас должна быть домашняя работа, и вы не можете связаться с профессором), безопаснее всего использовать 1 в качестве ведущего коэффициента в каждой строке.

Если ведущим коэффициентом в каждой строке является только ненулевое число в этом столбце, матрица называется уменьшенной эшелонированной строкой.

Матрица 3 × 5 в сокращенной форме эшелона строк.

Строковые формы эшелона обычно встречаются в линейной алгебре, когда вас иногда просят преобразовать матрицу в эту форму. Форма эшелона строк может помочь вам увидеть, что представляет собой матрица, а также является важным шагом к решению систем линейных уравнений.

Онлайн-калькулятор формы эшелона строк

Этот онлайн-калькулятор преобразует любую матрицу, и предоставляют операции со строками, которые помогут вам от шага к шагу.На следующем изображении (из калькулятора Old Dominion University Calculator показано, как матрица [01, 00, 59] приводится к форме эшелона строк с помощью двух простых операций со строками:

Back to Top.

Что такое форма сокращенного эшелона строк?

Уменьшенная форма эшелона строк — это тип матрицы, используемой для решения систем линейных уравнений. Форма пониженного ряда имеет четыре требования:

  • Первое ненулевое число в первой строке (, ведущая запись ) — это число 1.
  • Вторая строка также начинается с цифры 1, которая находится правее первой записи в первой строке. В каждом последующем ряду цифра 1 должна быть правее.
  • Начальная запись в каждой строке должна быть единственным ненулевым числом в ее столбце.
  • Любые ненулевые строки помещаются внизу матрицы.

Матрица 3 × 5 в сокращенной форме эшелона строк.

Преобразование матрицы в форму сокращенного эшелона строк

Любая матрица может быть преобразована в сокращенный вид эшелона строк с помощью метода, называемого исключением по Гауссу.Это особенно полезно для решения систем линейных уравнений. Большинство графических калькуляторов (например, TI-83) имеют функцию rref, которая преобразует матрицу в сокращенную форму эшелона строк. См. Эту статью на веб-сайте Университета штата Колорадо, где приведены инструкции по использованию TI-89 и TI-83 для расчета формы сокращенного эшелона строки.
Этот онлайн-калькулятор на веб-сайте Old Dominion University преобразует вводимую вами матрицу в сокращенную форму эшелона строк.

Для ручного расчета требуется знание элементарных операций со строками, а именно:

  1. Поменяйте местами одну строку с другой.
  2. Умножьте одну строку на ненулевую константу.
  3. Заменить одну строку на: одну строку плюс константу, умноженную на другую строку.

Кроме того, недостаточно просто знать правила, вы должны уметь взглянуть на матрицу и принять логическое решение о том, какое правило вы собираетесь использовать и когда. Вы пытаетесь преобразовать матрицу в сокращенный ряд строк, поэтому вам также нужно будет обратиться к четырем требованиям, приведенным в начале этой статьи. Если вам нужно вручную преобразовать матрицу в сокращенную форму эшелона строк, рекомендуется использовать один из приведенных выше калькуляторов, чтобы проверить свою работу.Фактически, если вы используете онлайн-калькулятор ODU, он даже предоставит вам операции со строками. На изображении ниже показано преобразование калькулятором матрицы [204,923]:

Вернуться к началу.

Что такое метод исключения Гаусса?

Метод исключения Гаусса — это способ найти решение системы линейных уравнений. Основная идея состоит в том, что вы выполняете математическую операцию над строкой и продолжаете, пока не останется только одна переменная. Например, некоторые возможные операции со строками:

  • Поменять местами любые два ряда
  • Сложите две строки вместе.
  • Умножить одну строку на ненулевую константу (например, 1/3, -1, 5)

Одновременно можно выполнять несколько операций со строками. Например, умножьте одну строку на константу, а затем добавьте результат к другой строке.

После этого цель состоит в том, чтобы получить матрицу в сокращенной форме эшелона строк, где ведущий коэффициент, 1, в каждой строке находится справа от ведущего коэффициента в строке над ней. Другими словами, вам нужно получить 1 в верхнем левом углу матрицы.В следующей строке должен быть 0 в позиции 1 и 1 в позиции 2. Это дает вам решение системы линейных уравнений.

Пример исключения Гаусса

Решите следующую систему линейных уравнений методом исключения Гаусса:

  • х + 5у = ​​7
  • -2x — 7y = -5

Шаг 1: Преобразуйте уравнение в форму матрицы коэффициентов . Другими словами, просто возьмите коэффициент для чисел и забудьте пока о переменных:

Шаг 2. Превратите числа в нижней строке в положительные, прибавив 2 раза первую строку:

Шаг 3: Умножьте вторую строку на 1/3.Это дает вам второй ведущий 1:

Шаг 4: Умножьте строку 2 на -5, а затем добавьте это к строке 1:

Вот и все!
В первой строке у вас x = -8, а во второй строке y = 3. Обратите внимание, что x и y находятся в тех же положениях, что и при преобразовании уравнения на шаге 1, поэтому все, что вам нужно сделать, это прочтите решение:

Вернуться к началу.

Что такое ранг матрицы?

Ранг матрицы равен количеству линейно независимых строк.Линейно независимая строка — это строка, которая не является комбинацией других строк.

Следующая матрица имеет две линейно независимых строки (1 и 2). Однако, когда в смесь добавляется третья строка, вы можете видеть, что первая строка теперь равна сумме второй и третьей строк. Следовательно, ранг этой конкретной матрицы равен 2, так как имеется только две линейно независимых строки.

Ранг матрицы всегда будет на меньше, чем — количество ненулевых строк или количество столбцов в матрице.Если все строки в матрице линейно независимы, матрица имеет полный ранг строки . Для квадратной матрицы она имеет полный ранг только в том случае, если ее определитель не равен нулю.

Определение ранга матрицы путем попытки определить только на глаз, сколько строк или столбцов являются линейно независимыми, может быть практически невозможно. Более простой (и, возможно, очевидный) способ — преобразовать в форму эшелона строк.

Как найти матрицу Рейтинг

Найти ранг матрицы просто, если вы знаете, как найти матрицу эшелона строк.Чтобы найти ранг любой матрицы:

  1. Найдите матрицу эшелона строк.
  2. Подсчитайте количество ненулевых строк.

Преобразование матрицы в форму эшелона строк.


Вышеупомянутая матрица была преобразована в форму эшелона строк с двумя ненулевыми строками. Следовательно, ранг матрицы равен 2.

Вы также можете найти отличный инструмент для конвертации на сайте Old Dominion University.

Вернуться к началу.

Список литературы

Эверитт, Б.S .; Скрондал А. (2010), Кембриджский статистический словарь, Cambridge University Press.
Гоник Л. (1993). Мультяшный справочник по статистике. HarperPerennial.
Серл, С. (2017). Матричная алгебра, полезная для статистики (серия Уайли по вероятности и статистике), 2-е издание. Вайли.

————————————————— —————————-

Нужна помощь с домашним заданием или контрольным вопросом? С Chegg Study вы можете получить пошаговые ответы на свои вопросы от эксперта в данной области.Ваши первые 30 минут с репетитором Chegg бесплатны!

Комментарии? Нужно опубликовать исправление? Пожалуйста, оставьте комментарий на нашей странице в Facebook .


[PDF] МЕТОДЫ ИСКЛЮЧЕНИЯ ГАУССА И ГАУССА-ИОРДАНА ДЛЯ РЕШЕНИЯ СИСТЕМЫ ЛИНЕЙНЫХ УРАВНЕНИЙ: СРАВНЕНИЯ И ПРИЛОЖЕНИЯ

1 Adenegn nd Aluko 97 Journal of Science nd Science Eduction, Ondo Vol. (), pp, 9 ноября, 0. Доступен онлайн t ISSN …

Аденеган и Алуко

97

Журнал науки и естественнонаучного образования, Ondo Vol.3 (1), pp. 97–105, 19 ноября 2012 г. Доступно на сайте http://www.josseo.org ISSN 0795135-3 © 2012

МЕТОДЫ УСТРАНЕНИЯ ПО ГАУССУ И ГАУСС-ИОРДАНУ ДЛЯ РЕШЕНИЯ СИСТЕМЫ ЛИНЕЙНЫХ УРАВНЕНИЙ: СРАВНЕНИЯ И ПРИЛОЖЕНИЯ Аденеган, Кехинде Эммануэль1 * и Алуко, Топе Мозес 1 1

Департамент математики, Педагогический колледж Адейеми, Ондо.

Аннотация: В данной статье исследуется сравнение методов Гаусса и Гаусса-Джордана для решения системы линейных уравнений.Обсуждалась различная терминология, рассматривались некоторые задачи в сочетании с вышеупомянутыми методами, используемыми при решении системы линейных уравнений. Из решенных задач очень примечательно было отмечено, что методы Гаусса и Гаусса-Жордана дают одинаковые ответы. Точно так же, когда одна и та же система поворачивается или частично поворачивается, легко получить одинаковые ответы. Это обязательно означает, что, поскольку одна и та же система линейных уравнений перестраивается, приводя к преобразованию ее матричной формы при очевидном изменении элемента строк, результирующие решения остаются теми же.В статье также явно показано, что методы исключения Гаусса / Гаусса и Гаусса-Жордана могут быть применены к различным системам линейных уравнений, возникающих в таких областях исследований, как физика, бизнес, экономика, химия и т. Д. Ключевые слова: матричный коэффициент, расширенная матрица, эшелон , Верхнетреугольная матрица, неопределенная система, вырожденное уравнение. ВВЕДЕНИЕ В линейной алгебре метод исключения Гаусса — это алгоритм решения системы линейных уравнений, нахождения ранга матрицы и вычисления обратной обратимой квадратной матрицы.Метод исключения Гаусса считается рабочей лошадкой вычислительной науки для решения системы линейных уравнений. Карл Фридрих Гаусс, великий математик 19 века, предложил этот метод исключения как часть своего доказательства определенной теоремы. Ученые-вычислители используют это «доказательство» как прямой вычислительный метод. Исключение Гаусса — это систематическое применение элементарных операций со строками к системе линейных уравнений с целью преобразования системы в верхнетреугольную форму.Когда матрица коэффициентов имеет верхнюю треугольную форму, мы используем обратную подстановку, чтобы найти решение. Метод исключения Гаусса помещает нули под каждой точкой поворота в матрице, начиная с верхней строки и работая на

вниз. Матрицы, содержащие нули под каждой точкой поворота, называются эшелонированной строкой. Процесс исключения Гаусса состоит из двух частей. Первая часть (прямое исключение) приводит данную систему либо к треугольной, либо к эшелонированной форме или приводит к вырожденному уравнению без решения, что указывает на то, что система не имеет решения.Это достигается за счет использования элементарных операций со строками. На втором этапе используется обратная подстановка для нахождения решения системы линейных уравнений. Другая точка зрения, которая оказывается очень полезной для анализа алгоритма, заключается в том, что метод исключения Гаусса вычисляет разложение матрицы. Операция с тремя элементарными строками, используемая при исключении Гаусса (умножение строк, переключение строк и добавление кратных строк к другим строкам), сводится к умножению исходной матрицы на обратимую матрицу слева.Первая часть алгоритма вычисляет LU-декомпозицию (разложение на нижнюю и верхнюю треугольную матрицу), а

* Автор-корреспондент. Электронная почта: [электронная почта защищена]

98

Journal of Science and Science Education, Ondo

вторая часть записывает исходную матрицу как произведение однозначно определенной обратимой матрицы и однозначно определенной уменьшенной матрицы строк. Метод Гаусса — Жордана представляет собой модификацию метода исключения Гаусса. Он назван в честь Карла Фридриха Гаусса и Вильгельма Джордана, потому что это разновидность метода исключения Гаусса, описанного Джорданом в 1887 году, в то время как метод исключения Гаусса помещает нули под каждым стержнем в матрице, начиная с верхней строки и двигаясь вниз, метод исключения Гаусса-Джордана идет следующим образом. сделайте шаг дальше, поместив нули выше и ниже каждой точки поворота.Каждая матрица имеет приведенную форму эшелона строк, и метод исключения Гаусса — Жордана гарантированно его найдет. Работа направлена ​​на исследование методов решения системы линейных уравнений с использованием методов исключения Гаусса и Гаусса — Жордана, сравнение и сопоставление двух методов и поиск применения этих методов в других областях исследования. ОБЗОР ЛИТЕРАТУРЫ Неудивительно, что начало матриц и детерминантов должно возникнуть в результате изучения линейных систем. Вавилоняне изучали проблемы, которые привели к одновременным линейным уравнениям, и некоторые из них сохранились в глиняных табличках, которые сохранились до наших дней.Китайцы между 200 г. и 100 г. до н. Э. Подошли к матрицам гораздо ближе, чем вавилоняне. Действительно, будет справедливо сказать, что текст из девяти глав по математическому искусству, написанный во времена династии Хань, дает первый известный пример матричных методов. Кардан в «Art Magna» (1545 г.) дает правило для решения системы двух линейных уравнений, которые он назвал регулярными de modo и которые называются матерью правил. Это правило дает то, что по сути является правилом Краммера для решения системы 2 x 2. Идея детерминанта появилась в Японии и Европе почти в одно и то же время, хотя Seki в Японии, безусловно, был опубликован первым.В 1683 году Секи написал метод решения диссимулированной проблемы, который содержит матричные методы, записанные в виде таблиц, точно так же, как китайские методы, описанные выше. Не имея ни одного слова, соответствующего «определителю», Секи все же ввел определители и дал общие методы их вычисления на основе примеров. Используя свой «определитель», Секи смог найти определители

матриц 2 x 2, 3 x 3, 4 x 4 и 5 x 5 и применил их для решения уравнений, но не системы линейных уравнений.В 1730-х годах Маклорен написал «Трактат по алгебре», хотя он был опубликован только в 1748 году, через два года после его смерти. Он содержит первые опубликованные результаты детерминантного доказательства правила Краммера для систем 2 x 2 и 3 x 3 и показывает, как будет работать случай 4 x 4. Краммер дал общее правило для систем nxn во введении к анализу алгебраических кривых (1750 г.). Это возникло из-за желания найти уравнение плоской кривой, проходящей через ряд заданных точек. В 1764 году Безу дал методы вычисления определителей, как это сделал Вандермонд в 1771 году.В 1772 году Лаплас заявил, что метод, предложенный Краммером и Безу, непрактичен, и в статье, в которой он изучал орбиты внутренних планет, он обсуждал решение системы линейных уравнений, фактически не вычисляя его с помощью определителей. Довольно удивительно, что Лаплас использовал слово «результирующий» для обозначения того, что мы теперь называем определителем. Удивительно, поскольку это то же слово, что использовал Лейбниц, однако Лаплас, должно быть, не знал о работе Лейбница. Лаплас дал расширение определителя, который теперь назван его именем.Жак Страм дал обобщение проблемы собственных значений в контексте решения системы обыкновенных дифференциальных уравнений. Фактически, понятие собственного значения снова появилось 80 годами ранее в работе О ‟Аламбера над системами линейных дифференциальных уравнений, изучающих движение струны с массой, прикрепленной к ней в различных точках. Первым, кто использовал термин «матрица», был Сильвестр в 1850 году. Сильвестр определил матрицу как продолговатое расположение терминов и рассматривал ее как нечто, что приводит к различным определителям из квадратных массивов, содержащихся в ней.Прожив в Америке и вернувшись в Англию в 1851 году, Сильвестр стал юристом и встретил Кэли, коллегу-юриста, который разделял его интерес к математике. Кэли быстро осознал важность концепции матрицы и к 1853 году Кейли опубликовал заметку, в которой впервые была дана обратная матрица. Фробениус в 1878 году написал важную работу по матрицам линейных подстановок и линейных форм, хотя, похоже, он не знал о работе Кэли. Фробениус в своей статье имел дело с эффективными формами

Аденегана и Алуко и не использовал термин «матрица».Однако он доказал важные результаты о канонических матрицах как представителях классов эквивалентности матриц. Он цитирует Кронекера и Вейерштрасса, которые рассмотрели частные случаи его результатов в 1874 и 1868 годах соответственно. Фробениус также доказал общий результат о том, что матрица удовлетворяет своему характеристическому уравнению. Эта статья Фробениуса 1878 года также содержала определение ранга матрицы, которое он использовал в своей работе над каноническими формами, и определение ортогональных матриц. Метод исключения Гаусса представлен в восьмой главе «Прямоугольный массив» китайского математического текста «Цзючжан Суаньшу» из девяти глав, посвященных математическому искусству.Его использование проиллюстрировано в восемнадцати задачах с двумя-пятью уравнениями. Первое упоминание о книге под этим названием датируется 179 годом до нашей эры, но некоторые ее части были написаны примерно 150 годом до нашей эры. Он был начат Лю Хуэем в 3 веке. Метод Европы проистекает из записок Исаака Ньютона. В 1670 году он написал, что во всех известных ему книгах по алгебре не было уроков по одновременному решению уравнения, которое затем предложил Ньютон. Кембриджский университет в конце концов опубликовал заметки как Арифметический университет в 1707 году, спустя много времени после того, как Ньютон оставил академическую жизнь.Эти заметки были широко инициированы, что сделало (то, что сейчас называется) методом исключения Гаусса стандартным уроком в учебниках алгебры к концу 18 века. Карл Фредерик Гаусс в 1810 году разработал обозначение для симметричного исключения, которое было принято в 19 веке профессиональными ручными компьютерами для решения нормальных уравнений задач наименьших квадратов. Гаусс разработал метод исключения Гаусса примерно в 1880 году и использовал его для решения задач наименьших квадратов в небесных вычислениях, а затем в вычислениях для измерения Земли и ее поверхности (раздел прикладной математики), связанных с измерением или определением формы Земли или с точным расположением точек. на земной поверхности называется геодезией.Хотя имя Гаусса связано с этой техникой последовательного исключения переменных из систем линейных уравнений. В течение многих лет метод исключения Гаусса считался частью развития геодезии, а не математики. Первое появление

99

исключения Гаусса-Джордана в печати было в справочнике или геодезии, написанном Вильгельмом-Джорданом. В области матричного анализа и линейной алгебры Карл Мейор (2000) пишет: «Несмотря на некоторую путаницу в отношении того, кому Джордан следует отдать должное за этот алгоритм, очевидно, что этот метод на самом деле был введен геодезистом по имени Вильгельм. Иордания (1842-1899), а не более известного математика Мари Эннемон Камилла Джордана (1838-1992), имя которой часто ошибочно связывают с этой техникой, но которому в остальном правильно приписывают другую важную тему в матричном анализе — Иорданию. Каноническая форма — самая заметная ».Определение правого Иордана — это еще не конец проблемы, по мнению А.С. Хаусхолд пишет в теории матриц в численном анализе (1964, стр. 141): «Метод Гаусса-Жордана, так называемый, кажется, был впервые описан Класеном (1888), поскольку его можно рассматривать как модификацию метода исключения Гаусса, имя Гаусс применяется правильно, но имя Джордана, по-видимому, связано с ошибкой, поскольку метод был описан только в третьем издании его Hanbuch der Vermes Sungskunde, подготовленном после его смерти ».Эти утверждения были рассмотрены S.C. Althoen и R. Mcluaghlin (1987). American Mathematical ежемесячно, 94, 130–142. Они пришли к выводу, что Хаусхолд был прав в отношении Класена и его публикации 1888 года, но ошибался в отношении Джордана, который был еще жив, когда в 1888 году вышло третье издание его книги. Они добавили, что «зародыш идеи» уже присутствовал во втором издании книги. 1877 г. (Эта запись была внесена Джоном Олдричем). Процесс исключения Гаусса состоит из двух частей. Первая часть (прямое исключение) приводит данную систему либо к треугольной, либо к эшелонированной форме, либо приводит к вырожденному уравнению без решения, что указывает на то, что система не имеет решения.Это достигается за счет использования элементарных операций со строками. На втором этапе используется обратная подстановка для нахождения решения системы линейных уравнений. Выражаясь эквивалентно для матриц, первая часть сводит матрицу к форме эшелона строк, используя элементарные операции со строками, тогда как вторая часть сводит ее к форме сокращенного эшелона строк. Другая точка зрения, которая оказывается очень полезной для анализа алгоритма, заключается в том, что метод исключения Гаусса

100

Journal of Science and Science Education, Ondo

вычисляет разложение матрицы.Три элементарные операции со строками, используемые в методе исключения Гаусса (умножение строк, переключение строк и добавление кратных строк к другим строкам), сводятся к умножению матрицы на обратимую матрицу слева. МЕТОДЫ ИСКЛЮЧЕНИЯ ГАУССА И ГАУССА-ИОРДАНА ДЛЯ РЕШЕНИЯ СИСТЕМЫ ЛИНЕЙНЫХ УРАВНЕНИЙ Система линейных уравнений (или линейная система) представляет собой набор линейных уравнений, включающих тот же набор переменных. Решение линейной системы — это присвоение значений переменной

x1, x2, x3,…, xn эквивалентно,  xi  i  1 такое, что каждое из уравнений выполняется. Множество всех возможных решений называется множеством решений. Решение линейной системы может вести себя одним из трех возможных способов: 1. Система имеет бесконечно много решений 2. Система имеет единственное единственное решение 3. Система не имеет решения Для трех переменных каждое линейное уравнение определяет плоскость в трехмерное пространство и множество решений — это пересечение этих плоскостей. Таким образом, набор решений может быть линией, отдельной точкой или пустым набором.Для переменных каждое линейное уравнение определяет гиперплоскость в n-мерном пространстве. Множество решений — это пересечение этих гиперплоскостей, которое может быть плоскостью любого размера. Набор решений для двух уравнений с тремя переменными обычно представляет собой линию. В общем, поведение линейной системы определяется соотношением между количеством уравнений и количеством неизвестных. Обычно система с меньшим количеством уравнений, чем неизвестных, имеет бесконечно много решений, такая система также известна как неопределенная система.Система с одинаковым количеством уравнений и неизвестных имеет единственное единственное решение. Система с большим количеством уравнений, чем неизвестных, не имеет решения. Существует множество методов решения линейных систем, которые включают: методы подстановки, методы исключения, методы обращения матриц, графические методы, методы Креммера, методы исключения Гаусса, методы исключения Гаусса-Джордана и т. Д. В этой статье будут рассмотрены n

методов исключения Гаусса и Гаусса-Жордана. Метод исключения Гаусса / Гаусса

Метод исключения Гаусса — это систематическое применение элементарных операций со строками к системе линейных уравнений с целью преобразования системы в верхнетреугольную форму.Когда матрица коэффициентов имеет верхнюю треугольную форму, мы используем обратную подстановку, чтобы найти решение. Общую процедуру исключения Гаусса можно резюмировать в следующих шагах: — Напишите расширенную матрицу для системы линейных уравнений. — Используйте элементарную операцию над {A / b}, чтобы преобразовать A в верхнюю треугольную форму. Если на диагонали расположен ноль, переключайте строки, пока в этом месте не окажется ненулевое значение. Если вы не можете этого сделать, остановитесь; система либо бесконечна, либо не имеет решения.- Используйте обратную замену, чтобы найти решение проблемы. Рассмотрим систему уравнений в матричной форме ниже.  a11 a12 a13  x1   b1      a21 a22 a23  x2    b2  a      31 a32 a33  x3   b3  Шаг 1: Запишите приведенное выше как расширенную матрицу  a11 a12 a13 b1     a21 a22 a23 b2  a   31 a32 a33 b3  Шаг 2: Исключите x1 из 2-го и 3-го уравнений aa, вычитая кратные m21  21 и m31  31 a11 a11 строки 1 из строк 2 и 3, получая эквивалентную систему:  a11 a12 a13 b1    2 2  2 a23  b2    0 a22  2 2   2 a33  b3    0 a32 Шаг 3: Исключите x2 из 3-го уравнения путем вычитания кратных m32 

a32

a22 строки 3, получая матричную систему:

2 2

строки 2 из

Adenegan and Aluko

 a11 a12 a13 b1     2 a23 2 b2 2   0 a22  3 3  0 a33  b3    0 Для иллюстрации, используя описанные выше шаги, решите систему x1  2 x2  3×3  3 2 x1  x2  x3  11 3×1  2 x2  x3  5 Это можно записать как 1 2 3  x1   3        2 1 1  x2    11   3 2 1  x   5    3    Расширенная матрица принимает вид  1 2 3 3      2 1 1 11   3 2 1 5    2 Теперь вычтите времена первый ряд из 1 3 второго ряда и умноженный на первый ряд из 1 третьего ряда, который дает;  1 2 3 3     0 5 5 5   0 4 10 14    4 4 Теперь вычтите, т.е.е. раз второй ряд №5 5 из третьего ряда. Тогда имеем  1 2 3 3     0 5 5 5   0 0 6 18    Обратите внимание, что в результате этих шагов матрица коэффициентов при x была сведена к треугольной матрице . Наконец, мы отделяем правый столбец обратно в исходное положение:  1 2 3  x1   3       0 5 5  x2    5   0 0 6  x   18    3    Тогда «обратной подстановкой», начиная с нижнего ряда, получаем: x1  2; x2  4; x3  3

101

Тот же пример и другие могут быть рассмотрены с использованием метода исключения Гаусса без поворота (как непосредственно решено выше) и с частичным поворотом, учитывая величину элементов в первом столбце i.е. 3  2  1, изменяя положение их строки в зависимости от величины, мы все равно получим тот же ответ. УСТРАНЕНИЕ ГАУССА-ИОРДАНА Исключение Гаусса-Жордана является модификацией исключения Гаусса. Мы снова преобразуем матрицу коэффициентов в другую матрицу, решение которой намного проще, и система, представленная новой расширенной матрицей, имеет тот же набор решений, что и исходная система линейных уравнений. В методе исключения Гаусса-Жордана цель состоит в том, чтобы преобразовать матрицу коэффициентов в диагональную матрицу, а нули вводятся в матрицу по одному столбцу за раз.Мы работаем над устранением элементов как выше, так и ниже диагональных элементов данного столбца за один проход через матрицу. Общую процедуру исключения Гаусс-Джордана можно резюмировать в следующих этапах: i. Напишите расширенную матрицу для системы линейных уравнений ii. Используйте операцию элементарной строки в расширенной матрице [A / b], чтобы преобразовать A в диагональную форму. Если по диагонали стоит ноль. Переключайте строки, пока в этом месте не окажется ненулевое значение. Если вы не можете этого сделать, остановитесь; система либо бесконечна, либо не имеет решения.iii. Разделив диагональные элементы и элемент правой стороны в каждой строке на диагональные элементы в строке, сделайте все диагональные элементы равными единице. Дана матрица системы уравнений 4 x 4 вида:  a11 a12 a13 a14  x1   b1      a21 a22 a23 a24  x2   b2   a31 a32 a33 a34   x3   b3        a41 a42 a43 a44   x4   b4  Шаг 1: Запишите приведенное выше как расширенную матрицу, у нас есть

102

Journal of Science and Science Education, Ondo

 a11   a21  a31   a41

b1   b2  b3   b4  Шаг 2. Исключите x1 из 2-го, 3-го и 4-го уравнения, вычтя из m21 кратные a31 a21 a41 строки 1. , m31  и m41  a11 a11 a11 2,3 и 4 соответственно, что дает a13 a14 b1   a11 a12   2  2  2 a23 a24  b2 2   0 a22   2 2  2 a33  a34  b3 2   0 a32  0 a  2 a  2 a  2 b  2  42 43 44 4   a12 a22 a32 a42

a13 a23 a33 a43

a14 a24 a34 a44

Шаг 3: исключить x2 из 1-го, 3-го и 4-го уравнения путем вычитания кратных 2 2 a32  a12 a42  m12   2, m32   2, m42   2  ряда 2 а22 a22 a22 из строк 1, 3 и 4, чтобы получить 3 3  a113 0 a13  a14  b13    2 2 2  0 a22  a23  a24  b2 2    3 3 0 a33  a34  b33   0   0 a433 a443 b43   0 Шаг 4: Удалите x3 из 1-го, 2-го и 4-го уравнения путем вычитания кратных 3 3 3 a  a  a  m13  133, m23  233, m43  433 строки 3 из a33 a33 a33 строки 1, 2 и 4, получая 4 4  a11 4 0 0 a14  b1    4 4 4  0 a22  0 a24  b2     0 a333 a343 b33   0   0 0 a44 4 b4 4   0 Из которого мы окончательно решаем для x1, x2, x3 и x4 из полученных совместных уравнений сверху.Для сравнения решим задачу (3.1) методом исключения Гаусса-Жордана. т.е.

 1 2 3  x1   3        2 1 1  x2    11   3 2 1  x   5    3    Расширенная матрица становится  1 2 3 3     2 1 1 11   3 2 1 5    Теперь мы удалим x1 из 2-го и 3-го уравнения, вычитая кратное a31 a21 m21   2, m31   3 ряда 1 из ряда 2 a11 a11 и 3 соответственно производя.  1 2 3 3     0 5 5 5   0 4 10 14    st Исключите x2 из 1 и 3 уравнения, вычитая несколько aa 2 4 m12  12  и m32  32  строки 2 из a22 5 a22 5 строки 1 и строки 3 Мы имеем  1 0 1 5     0 5 5 5   0 0 6 18    st Исключим x3 из 1 и 2 уравнения с помощью вычитая несколько aa 1 5 m13  13  и m23  23  строки 3 из a33 6 a33 6 строки 1 и строки 3.1 0 0 2     0 5 0 20   0 0 6 18   

Аденеган и Алуко. В итоге получается  x1   2       x2    4   x   3   3   Для дополнительной иллюстрации рассмотрим следующую систему уравнений с использованием исключения Гаусса-Жордана (i) без поворота (ii) с частичным поворотом  1 1 1 1  x1   1        1 1 1 1   x2    2   2 4 3 5   x3   2      3 1 1 1   x4   1  Представим вышеуказанное в форме расширенной матрицы (i) без поворота  1 1 1 1 1     1 1 1 1 2   2 4 3 5 2      3 1 1 1 1  Теперь мы исключим x1 из 2-го, 3-го и 4-го уравнения, вычитая кратные aaa m21  21  1, m31  31  2 и, m41  41  3 из a11 a11. a11 строка 1 из строки 2, 3 и 4 соответственно, получая  1 1 1 1 1     0 2 2 0 1   0 6 1 7 4    0 4  2 4  4   Исключите x2 из уравнений (1), (3) и (4), вычтя кратные 2 a32 6 a12 a42  1 m12   2 , m32   2  3 и, m42   2  2 а22 а22  2 a22 строки 2 из строки 1, 3 и 4 соответственно, что дает  1 0 0 1 3  2   0 2 2 0 1  0 0 7 7  7    0 0 2 4 6    Исключите x3 из уравнений (1), (2) и (4) путем вычитания кратных

103

a13 0 aa 2 2   0, m23  23  и, m43  43  a33 7 a33 7 a33 7 строки 3 из строк 1, 2 и 4, чтобы получить  1 0 0 1 3  2   0 2 0 2 1  0 0 7 7  7    0 0 0 2 4    Исключить x4 из уравнений (1), (2) и (3) и вычесть кратные aaa 1 7 m14  14  , m24  24  1 и, m34  34  a44 2 a44 a44 2 строки 4 из строк 1, 2 и 3, получая  1 0 0 0 1  2   0 2 0 0 3  0 0 7 0  7    0 0 0 2 4      1  R2  1 R2  1 0 0 0 2  2  0 1 0 0 3  R3  1 R3 2   7 0 0 1 0 1  0 0 0 1  R4  1 R4  2 2    x1  1, x2  3, x3  1 и x4  2 2 2 Следовательно,  1   x1   2       x2    3 2   x3      1   x4   2    4  (ii) С частичным поворотом  2 2 Так как 3  2  1  1, то имеем те же ans мы следовали тому же процессу.m13 

ПРИМЕНЕНИЕ МЕТОДОВ ГАУССА И ГАУССЙОРДАНА ДЛЯ РЕШЕНИЯ СИСТЕМЫ ЛИНЕЙНЫХ УРАВНЕНИЙ Методы Гаусса и Гаусса-Жордана решения системы линейных уравнений могут быть использованы в различных предметных областях и используются для решения для неизвестных. С n уравнениями максимально возможное количество неизвестных, которые можно решить, равно n. сложности возникают, когда число неизвестных

104

Journal of Science and Science Education, Ondo

не равно количеству уравнений в системе.Линейная система уравнений с большим или меньшим количеством уравнений, чем неизвестных, встречается довольно часто. Когда уравнений меньше, чем неизвестных, невозможно решить все неизвестные алгебраически. Из-за этого обычно существует бесконечное множество решений для каждой переменной (трудно сузить круг). Когда уравнений больше, чем неизвестных (переопределенная система), решений обычно нет. Это происходит потому, что может быть решение, которое удовлетворяет двум из трех уравнений, но не обязательно всем трем.Единственный способ найти решение переопределенной системы (например, системы с тремя уравнениями и двумя неизвестными) — это если все уравнения эквивалентны, если два из трех уравнений эквивалентны, а другое уравнение пересекает друг друга в точке единственная точка. Некоторые из областей, в которых может применяться метод Гаусса и Гаусса-Джордана, — это бизнес и экономика, физика, биология, химия, инженерия и т. Д. Применение в бизнесе Методы Гаусса и Гаусса-Джордана для решения линейных систем могут быть использованы в бизнес-образовании.На самом деле бизнес и экономика взаимосвязаны. Швейная промышленность Bolade Nig-Limited производит три стиля рубашек. Для каждой рубашки требуются услуги трех универмагов, указанных ниже. СТИЛИ A B C Раскрой 0,2 0,4 0,3 Шитье 0,3 0,5 0,4 Упаковка 0,1 0,2 0,1 На раскрой, шитье и упаковку доступно максимум 1160, 1560 и 480 часов. Вы обязаны: i. Сформулируйте информацию в виде линейных уравнений. II. Используя методы исключения Гаусса и Гаусса-Джордана, найдите количество рубашек, которое необходимо производить каждую неделю, чтобы завод работал на полную мощность.Пусть x представляет стиль A Пусть y представляет стиль B Пусть z представляет стиль C

0,2 ​​x  0,4 y  0,3z  1160 0,3x  0,5 y  0,4 z  1560

i   ii   iii 

0,1x  0,2 y  0,1z  480 Решение дает x = 1200, y = 800 и z = 2000 (все в единицах)

Применение в экономике Методы Гаусса и Гаусса-Жордана могут использоваться для определения равновесной цены и количества поставляться на данный рынок. Допустим, есть два продукта: апельсиновый сок и вода, которые взаимосвязаны.Пусть P1 и q1 представляют цену и количество, требуемые соответственно для продукта 1 (апельсиновый сок), а P1 и q2 представляют то же самое для продукта 2 (вода). Предложение спроса Продукт 1: P1  2000 — 3q1 — 2q2, q1  100  2q1  q2

Продукт 2: P2  2800 — q1 — 4q2, q2  200  3q1  2q2 Для достижения равновесия обе цены выражения должны совпадать, поэтому получаются следующие уравнения. Продукт 1: 2000 — 3q1 — 2q2 = 100  2q1  q2

Результат 2: 2800 — q1 — 4q2 = 200 3q1  2q2 Приведенные выше выражения после рефрейминга и решения дают q1 = 200, q2 = 300.Следовательно, P1 = N800, P2 = N1,400. Поиск равновесной цены и количества на рынке очень важно для экономиста, поскольку за пределами этих значений ничего не будет продано с прибылью. Приложения в физике Могут возникать или формулироваться аналогичные проблемы, описанные выше, и могут быть легко применены методы Гаусса и Гаусса-Жордана. ЗАКЛЮЧЕНИЕ В результате исследования был сделан вывод, что при выполнении вычислений вручную метод Гаусса-Жордана более предпочтителен, чем вариант исключения Гаусса, поскольку он позволяет избежать необходимости обратной подстановки.Кроме того, очень примечательно было отмечено, что и метод исключения Гаусса, и метод исключения Гаусса-Жордана дали одинаковые ответы для каждого рабочего примера, и оба с поворотом и с частичным поворотом в равной степени дали одинаковые ответы для

Аденеган и Алуко. Это обязательно означает, что, поскольку система линейных уравнений остается той же самой, несмотря на то, что уравнения переупорядочиваются, что приводит к преобразованию ее матричной формы, поскольку элементы строк ‟, очевидно, изменились, результирующие решения остаются теми же.Важность метода исключения Гаусса и Гаусса-Джордана невозможно переоценить из-за его значимости для различных областей исследований (чистая наука, то есть физика, биология, химия, математика и т. Д., Социальные науки, то есть экономика, география, бизнес-образование и т. Д.). ССЫЛКИ Adeola. Т.А. (2009): Исследование решения системы линейных уравнений, неопубликованная проектная работа, Педагогический колледж Адейеми. Ондо.

105

Андерсон, Дж. П. (1982): Математический анализ и приложения к бизнесу и экономике, 3-е изд.PEP Нью-Йорк. Антон, Х. (2005): Элементарная линейная алгебра (версия приложения — 9-е издание) Willey International, Лондон. http://www.purplemath.com/./systlin6.htm (2010 г.). http://www.matrixanalysis.com/downloadchapter.html (2007 г.). Илори, С.А.; Акиниэле О. (1986): Элементарная абстрактная и линейная алгебра, Издательство Ибаданского университета, Ибадан. Мэр, C.D. (2001): Инженерная математика 5-е изд. Энтони Роу Лтд. Чиппенхем, Уилтшир. Сеймур, Л. (1981): теория и проблема линейной алгебры — ISBN 0-07-99012-3.Уэббер, Дж. П. (1982): Математический анализ, 4-е изд; Институт Харпира; Лондон

(сокращенный) Калькулятор формы эшелона строк

Добро пожаловать в калькулятор формы сокращенного эшелона строки (или для краткости калькулятор rref ), где мы решим систему уравнений по вашему выбору, используя сокращение строк матрицы и элементарные операции со строками. Кроме того, мы даем вам возможность выбрать, хотите ли вы использовать сокращенную версию или нет. В зависимости от вашего выбора, наш инструмент можно рассматривать как калькулятор исключения Гаусса-Джордана (с первым вариантом) или калькулятор исключения Гаусса .Более того, если в вашей системе бесконечное количество решений, наш калькулятор rref даже подскажет, как они выглядят!

Что такое система уравнений?

Помните все те математические сценарии, которые пытаются имитировать реальную жизнь? Как маленькая девочка спрашивает, сколько ей лет, если через десять лет ее мама будет вдвое старше, чем тогда? Знаете, только ваши повседневных разговоров и повседневные проблемы . Что ж, уравнения — это то, что мы используем для их решения.

Всякий раз, когда у нас есть какое-то значение, которое мы не знаем (например, возраст маленькой девочки), но мы знаем, что оно должно удовлетворять определенному свойству (например, быть вдвое большим, чем какое-либо другое число), мы описываем эту связь, используя уравнения .Мы обозначаем неизвестное нам значение символом, который мы называем переменной . Затем мы записываем то, что нам известно об этом, с помощью математических символов и операций, таких как сложение, вычитание, умножение или деление. Полученное выражение называется уравнением .

Если у нас есть несколько уравнений и мы хотим, чтобы все они удовлетворялись одним и тем же числом, то мы имеем дело с системой уравнений . Обычно они имеют в общей сложности более одной переменной, и наиболее распространенные математические задачи включают в себя то же количество уравнений, что и переменных.Например, предположим, что мать нашей маленькой девочки говорит нам, что она в три раза старше своей дочери . Теперь мы знаем, откуда взялось это остроумие … В любом случае, мы можем преобразовать это новое заявление мамы в уравнение. Вместе с предыдущим они образуют систему двух уравнений с двумя переменными: возрастом девочки и возрастом матери.

Операции элементарных строк

Давайте попробуем увидеть , как наш калькулятор сокращенной формы эшелона строк видит систему уравнений .Возьмем этот пикантный пример:

.

Не волнуйтесь, мы не вернулись в детский сад (хотя и не прочь вздремнуть), мы все еще работаем с системами уравнений. Картинка выше может не выглядеть так, но на самом деле — это всего лишь . Мы настолько привыкли видеть переменные, как x или y , что склонны забывать, что это просто символ неизвестного нам значения. И здесь у нас есть сочный лимон, хрустящее яблоко и сладкий банан, и все они представляют собой числа, которых мы еще не знаем.Для простоты обозначения обозначим их как x , y и z соответственно. Таким образом, мы можем написать эквивалентную систему уравнений :

х + у + г = 32 ,

y + y - x = 25 и

z + z - y = 16 .

Теперь, когда мы смотрим на это, средняя школа убила часть нашего воображения, не так ли? Тем не менее, теперь мы можем легко упростить систему , добавив вместе одинаковые символы в последовательные уравнения и записав переменные, которые появляются в них в алфавитном порядке .Например, во втором уравнении y + y - x = 25 , мы можем сложить y вместе, чтобы получить 2y - x = 25 (поскольку у нас было две копии y ). Затем мы переворачиваем переменную x в начало, чтобы создать алфавитный порядок (не забудьте, что для укажите число рядом с ним ) и получаем -x + 2y = 25 . Всего получаем

х + у + г = 32 ,

-x + 2y = 25 и

-y + 2z = 16 .

Калькулятор rref использует исключение Гаусса-Жордана и исключение Гаусса , и оба используют так называемое сокращение строки матрицы . Это, в свою очередь, зависит от операций с элементарной строкой , а именно:

  1. Вы можете поменять местами любые два уравнения.
  2. Вы можете умножить любое уравнение на ненулевое постоянное число.
  3. Вы можете добавить ненулевое кратное любому уравнению к другому уравнению.

Под « вы можете » мы подразумеваем то, что система, которую вы получите в результате этих операций, будет на эквивалента той, с которой вы начали.Это означает, что у двух будет точно таких же решений .

Например, мы могли бы умножить первое уравнение, скажем, на -3 :

-3x - 3y - 3z = -96

-x + 2y = 25

-y + 2z = 16 ,

и добавьте две копии второго уравнения к третьему:

-3x - 3y - 3z = -96

-x + 2y = 25

-y + 2z + 2 * (-x + 2y) = 16 + 2 * 25 ,

, что равно

-3x - 3y - 3z = -96

-x + 2y = 25

-2x + 3y + 2z = 66 .

Элементарные операции со строками не изменили набор решений для нашей системы . Не верите нам? Продолжайте, введите первую и последнюю систему в калькулятор сокращенной формы эшелона строк и посмотрите, что вы получите. Мы будем ждать вас, но когда вы вернетесь, ждите: «, как мы вам сказали, ».

Исключение Гаусса-Джордана против исключения Гаусса

Мы можем использовать сокращение строк матрицы, которое мы упомянули в разделе выше, для более практических целей, чем просто для развлечения с умножением уравнений на случайные числа.Да ладно, мы действительно повеселились, не так ли?

Как вы могли догадаться, легче иметь дело с одной переменной, чем с несколькими из них, так почему бы не попробовать удалить некоторые из них ? Предположительно, это (но на немецком языке) было мышлением Карла Фридриха Гаусса , математика, стоящего за так называемым исключением Гаусса . Это алгоритмическая процедура, которая преобразует систему уравнений в очень простую для работы форму. Идея, лежащая в основе этого (пожалуйста, прочтите следующие инструкции с немецким акцентом 18-го века ):

  1. Возьмите уравнение с первой переменной в нем и поместите эту строку как первую в вашей системе .
  2. Используйте элементарные операции со строками в первом уравнении, чтобы исключить все вхождения первой переменной во всех остальных уравнениях.
  3. Возьмите уравнение (отличное от первого) со второй переменной в нем и поместите его как вторую в системе .
  4. Используйте элементарные операции со строками во втором уравнении, чтобы исключить все вхождения второй переменной во всех последующих уравнениях.
  5. Повторите для последующих переменных , пока у вас не закончатся уравнения, переменные или самодисциплина для завершения упражнения.

Система, которую мы получаем в итоге, называется в рядном эшелоне формы . « Так что же представляет собой калькулятор уменьшенный в форме уменьшенного ряда? » Как удобно с вашей стороны спросить! Вот тут-то и появляется — метод исключения Гаусса-Джордана . Это немного улучшенная версия предыдущего алгоритма, впервые выполненная Камиллой Джордан . Французскому математику потребовалось несколько десятилетий, чтобы задать фундаментальный вопрос: « Что, если в конце концов мы разделим каждую строку на ее первое число? » Mind = blown.

Другими словами, надстройка исключения Гаусса-Джордана дает нам дополнительный шаг в алгоритме :

  1. Разделите каждое уравнение на коэффициент первой переменной , встречающейся в этой строке.

Система, которую мы получаем с обновленной версией алгоритма, называется в сокращенном эшелоне строк формы . Преимущество такого подхода состоит в том, что в каждой строке перед первой переменной будет стоять коэффициент 1 вместо чего-то сложного, например, 2 .Однако это ускоряет вычисления, и, как мы знаем, каждая секунда ценится.

Пора привести пример, не так ли?

Пример: использование калькулятора сокращенной ступенчатой ​​формы

Вспомните систему уравнений, которая была у нас во втором разделе , но та, что была прямо перед тем, как мы начали играть с элементарными операциями со строками:

х + у + г = 32 ,

-x + 2y = 25 и

-y + 2z = 16 .

Прежде, чем мы перейдем к пошаговым вычислениям, давайте быстро скажем несколько слов о , как мы можем ввести такую ​​систему в наш калькулятор формы сокращенного эшелона . Прежде всего, у нас есть три строки в системе, поэтому нам нужно сообщить об этом калькулятору вверху, в поле количества уравнений. Это покажет нам символическую картину произвольной системы трех линейных уравнений.

Нам нужно определить, какое число соответствует какому символу из калькулятора rref.На рисунке первое уравнение имеет символы a₁ , b₁ , c₁ и d₁ , которые находятся соответственно рядом с x , с y , с z и справа. сторона знака = . Это числа, которые мы ищем в нашей системе. Глядя на первое из наших уравнений, мы определяем, что a₁ = 1 , b₁ = 1 , c₁ = 1 и d₁ = 32 ( помните, что отсутствие числа перед переменной означает, что коэффициент равно 1 ).

Аналогично для следующих двух строк получаем a₂ = -1 , b₂ = 2 , c₂ = 0 , d₂ = 25 и a₃ = 0 , b₃ = -1 , c₃ = 2 , d₃ = 16 ( помните, что если уравнение не имеет какой-либо переменной, то коэффициент рядом с этой переменной равен 0 ). Если вы введете все эти данные в калькулятор формы сокращенного эшелона строк, , вы получите спойлер с ответом .Также обратите внимание, что наш калькулятор rref не допускает нелинейных (например, квадратичных) уравнений .

Теперь мы будем следовать инструкциям по уменьшению строки матрицы, заданным , методом исключения Гаусса , чтобы преобразовать ее в форму эшелона строк. Наконец, мы сделаем дополнительный шаг от до исключения Гаусса-Джордана, чтобы превратить его в сокращенную версию, которая используется по умолчанию в калькуляторе rref.

Согласно алгоритму, мы начинаем с , выбираем уравнение с первой переменной (в нашем случае это x ) и помещаем его в верхнюю строку.Обратите внимание, что наша система уже находится в этой форме, поэтому нам не нужно ничего менять. Затем, , мы используем первое уравнение, чтобы исключить x из двух других строк . Обратите внимание, что нам нужно иметь дело только со вторым, поскольку в третьем уравнении нет x . Чтобы избавиться от -x в средней строке, нам нужно добавить к этому уравнению кратное первому уравнению, чтобы числа x компенсировали друг друга. Поскольку -x + x = 0 , нам нужно иметь x с коэффициентом 1 в том, что мы добавляем во вторую строку.К счастью, это именно то, что мы имеем в верхнем уравнении. Следовательно, мы добавляем первую строку ко второй , чтобы получить

-x + 2y + (x + y + z) = 25 + 32 ,

, что равно

3у + z = 57 .

Вместе с двумя другими уравнениями это дает

х + у + г = 32

3y + z = 57

-y + 2z = 16 .

Отлично! Теперь у нас есть две последние строки без x в них .Правда, второе уравнение получило z , которого раньше не было, но это просто цена, которую мы должны заплатить.

Теперь нам нужно что-то сделать с и в последнем уравнении, и мы будем использовать для этого вторую строку. Однако с это будет не так просто, как в прошлый раз — в нашем распоряжении 3y и -y , с которыми нужно справиться. Что ж, инструменты, которые они нам дали, будут делать.

Чтобы исключить -y из третьего уравнения, нам нужно получить y (т.е.е., y с коэффициентом 1 ) от второго, так как -y + y = 0 . Чтобы получить его с 3y , достаточно разделить его на 3 . Другими словами, на языке сокращения строк матрицы мы добавим к нижней строке кратное 1/3 (эквивалентно делению на 3 ) второго уравнения. Это дает

-y + 2z + (1/3) * (3y + z) = 16 + (1/3) * 57 .

Обратите внимание на то, что 1/3 также появился на правой стороне с 57 .После упрощения это дает

(7/3) z = 35 ,

, что вместе с двумя другими уравнениями равно

х + у + г = 32 ,

3y + z = 57 ,

(7/3) z = 35 .

Вуаля! То есть форма эшелона строки, заданная методом исключения Гаусса . Обратите внимание, что такие системы можно получить в нашем калькуляторе rref, ответив «» на вопрос, показывать ли сокращенную форму в верхней части калькулятора.

Чтобы получить сокращенную форму эшелона строк, мы следуем шестому шагу, упомянутому в разделе выше — , мы делим каждое уравнение на коэффициент его первой переменной . Это означает, что нам нужно разделить первую строку на 1 (коэффициент x ), вторую на 3 (коэффициент y ) и третью на 7/3 (коэффициент из z ). Это дает

х + у + г = 32 ,

y + (1/3) z = 19 ,

г = 15 ,

и знаменуют конец алгоритма исключения Гаусса-Жордана .Мы можем получить такие системы в нашем калькуляторе формы сокращенного эшелона строк, ответив « Да, » на главный вопрос (как это делается по умолчанию).

Обратите внимание, что теперь легко найти решение для нашей системы . Из последней строки мы знаем, что z = 15 , поэтому мы можем подставить его во второе уравнение, чтобы получить

.

y + (1/3) * 15 = 19 .

Из этого мы получаем y = 14 , и мы можем заменить это и z = 15 в первую строку, чтобы получить

х + 14 + 15 = 32 ,

, что дает x = 3 .Возвращаясь к картинке, с которой мы начали, это означает, что лимон равно 3 , яблоко равно 14 , и банан 15 . Теперь, когда мы знаем наши фрукты, мы можем нарезать их и съесть с блинами. Мы это заслужили .

1.2 Гауссово исключение. — ppt видео онлайн скачать

Презентация на тему: «1.2 Гауссовское исключение» — стенограмма презентации:

ins [data-ad-slot = «4502451947»] {display: none! important;}} @media (max-width: 800px) {# place_14> ins: not ([data-ad-slot = «4502451947»]) {display: none! important;}} @media (max-width: 800px) {# place_14 {width: 250px;}} @media (max-width: 500 пикселей) {# place_14 {width: 120px;}} ]]>

1 1.2 Исключение Гаусса

2 Формы эшелона Эта матрица, которая имеет следующие свойства, представлена ​​в сокращенной форме строки-эшелона (Пример 1, 2). 1. Если строка не состоит полностью из нулей, то первое ненулевое число в строке — 1. Мы называем это лидером 1. 2. Если есть какие-либо строки, которые полностью состоят из нулей, то они группируются вместе в нижняя часть матрицы. 3. В любых двух последовательных рядах, которые не состоят полностью из нулей, выноска 1 в нижнем ряду располагается дальше вправо, чем выноска 1 в более высоком ряду.4. В каждом столбце, содержащем выноску 1, повсюду нули. Матрица, которая имеет первые три свойства, называется строчно-эшелонированной (пример 1, 2). Матрица в редуцированной форме строки-эшелона обязательно должна иметь форму строки-эшелона, но не наоборот.

3 Пример 1 Форма рядов-эшелонов и сокращенных рядов-эшелонов

4 Пример 2 Подробнее о формах Row-Echelon и Reduced Row-Echelon
Все матрицы следующих типов представлены в форме row-echelon (любые действительные числа заменены на *.): Все матрицы следующих типов представлены в сокращенной форме строки-эшелона (любые действительные числа заменены на *.):

5 Пример 3 Решения четырех линейных систем (a)
Предположим, что расширенная матрица для системы линейных уравнений была приведена с помощью строковых операций к данной сокращенной форме «строка-эшелон». Решите систему. Решение (а) соответствующей системы уравнений:

6 Пример 3 Решения четырех линейных систем (b1)
Решение (b) 1.Соответствующая система уравнений: свободные переменные ведущие переменные

7 Пример 3 Решения четырех линейных систем (b2)
2. Мы видим, что свободной переменной может быть присвоено произвольное значение, например t, которое затем определяет значения ведущих переменных. 3. Существует бесконечно много решений, и общее решение дается формулами

8 Пример 3 Решения четырех линейных систем (c1)
Решение (c) Четвертая строка нулей приводит к уравнению, не накладывает никаких ограничений на решения (почему?).Таким образом, мы можем опустить это уравнение.

9 Пример 3 Решения четырех линейных систем (c2)
Решение (c) Решение для ведущих переменных в терминах свободных переменных: свободной переменной может быть присвоено произвольное значение, существует бесконечно много решений, и общее решение имеет вид заданные формулами.

10 Пример 3 Решения четырех линейных систем (d)
Решение (d): последнее уравнение в соответствующей системе уравнений: Поскольку это уравнение не может быть выполнено, у системы нет решения.

11 Методы исключения (1/7)
Мы дадим пошаговую процедуру исключения, которую можно использовать для приведения любой матрицы к сокращенной форме ряда строк.

12 Методы исключения (2/7)
Шаг 1. Найдите крайний левый столбец, который не состоит полностью из нулей. Шаг 2. Поменяйте местами верхнюю строку с другой строкой, чтобы ненулевой элемент находился в верхней части столбца, найденного на шаге 1.Крайний левый ненулевой столбец 1-я и 2-я строки в предыдущей матрице поменялись местами.

13 Методы исключения (3/7)
Шаг 3. Если запись, которая сейчас находится наверху столбца, найденного на шаге 1, представляет собой a, умножьте первую строку на 1 / a, чтобы ввести начальную единицу. Шаг 4. Добавьте подходящие числа, кратные верхнему ряду, к нижеследующим строкам, чтобы все входы ниже первой единицы стали нулями. Первая строка предыдущей матрицы была умножена на 1/2.-2 раза первая строка предыдущей матрицы была добавлена ​​к третьей строке.

14 Методы ликвидации (4/7)
Шаг 5. Теперь закройте верхнюю строку матрицы и начните снова с Step1, примененного к оставшейся подматрице. Продолжайте таким образом, пока вся матрица не будет в виде ряда строк. Крайний левый ненулевой столбец в подматрице Первая строка в подматрице была умножена на -1/2, чтобы ввести начальную единицу.

15 Методы исключения (5/7)
Шаг 5 (продолжение) -5 раз 1-я строка подматрицы была добавлена ​​ко 2-й строке подматрицы, чтобы ввести ноль ниже ведущего 1. Верхняя строка подматрицы была покрыта , и мы снова вернулись к Step1. Крайний левый ненулевой столбец в новой подматрице Первая (и единственная) строка в новой подматрице была умножена на 2, чтобы ввести начальную единицу. Теперь вся матрица находится в виде эшелона строк.

16 Методы исключения (6/7)
Шаг 6. Начиная с последней ненулевой строки и двигаясь вверх, добавьте подходящие числа, кратные каждой строке, к строкам выше, чтобы ввести нули над ведущими единицами. 7/2 раза 3-я строка предыдущей матрицы была добавлена ​​ко 2-й строке. -6 раз прибавляли 3-й ряд к 1-му ряду. 5 раз 2-й ряд прибавляли к 1-му ряду. Последняя матрица приведена в сокращенном виде поэтапно.

17 Методы исключения (7/7)
Шаги 1 ~ Шаг 5: вышеупомянутая процедура создает форму строки-эшелон и называется исключением по Гауссу. Шаг 1 ~ Шаг 6: описанная выше процедура производит сокращенную форму строки-эшелон и называется исключением Гаусса-Жордана. Каждая матрица имеет уникальную сокращенную форму строки-эшелон, но форма строки-эшелона данной матрицы не уникальна.

18 Пример 4 Исключение Гаусса-Жордана (1/4)
Решение методом исключения Гаусса-Жордана Решение: Расширенная матрица для системы

19 Пример 4 Исключение Гаусса-Джордана (2/4)
Добавление -2 раза 1-й строки ко 2-й и 4-й строкам дает Умножение 2-й строки на -1, а затем прибавление -5 раз новой 2-й строки к 3-й строке и -4 раза новый 2-й ряд к 4-му ряду дает

20 Пример 4 Исключение Гаусса-Жордана (3/4)
Меняя местами 3-ю и 4-ю строки, а затем умножая 3-ю строку результирующей матрицы на 1/6, получаем форму «строка-эшелон».Добавление -3 раза 3-й строки ко 2-й строке и затем 2-кратное прибавление 2-й строки итоговой матрицы к 1-й строке дает уменьшенную форму строки-эшелона.

21 год Пример 4 Исключение Гаусса-Жордана (4/4)
Соответствующая система уравнений — Решение. Расширенная матрица для системы: Мы назначаем свободные переменные, а общее решение дается формулами:

22 Обратная подстановка Иногда предпочтительнее решать систему линейных уравнений с помощью исключения Гаусса, чтобы привести расширенную матрицу в форму строки-эшелон, не переходя полностью к сокращенной форме строки-эшелона.Когда это будет сделано, соответствующая система уравнений может быть решена с помощью метода, называемого обратной подстановкой. Пример 5

23 Пример 5 ex4, решенный с помощью обратной подстановки (1/2)
Из вычислений в Примере 4, форма строки-эшелон из расширенной матрицы должна решить соответствующую систему уравнений Step1. Решите уравнения для главных переменных.

24 Example5 ex4 решено обратной заменой (2/2)
Step2.Начиная с нижнего уравнения и двигаясь вверх, последовательно подставляйте каждое уравнение во все уравнения над ним. Подставляем x6 = 1/3 во 2-е уравнение. Подставляем x3 = -2 x4 в 1-е уравнение. Шаг 3. Назначьте свободные переменные, общее решение дается формулами.

25 Пример 6 Исключение Гаусса (1/2)
Решается методом исключения Гаусса и обратной подстановки.(ex3 из Раздела 1.1) Решение Мы преобразуем расширенную матрицу в форму потока-эшелона. Система, соответствующая этой матрице, имеет вид

26 Пример 6 Исключение Гаусса (2/2)
Решение Решение для ведущих переменных Подставление нижнего уравнения в приведенное выше Подставление 2-го уравнения в верхнее

27 Однородные линейные системы (1/2)
Система линейных уравнений называется однородной, если все постоянные члены равны нулю; то есть система имеет вид: каждая однородная система линейного уравнения непротиворечива, поскольку вся такая система имеет в качестве решения.Это решение называется тривиальным решением; если есть другие решения, они называются нетривиальными решениями. Есть только две возможности для ее решения: Система имеет только тривиальное решение. Система имеет бесконечно много решений помимо тривиального решения.

28 год Однородные линейные системы (2/2)
В частном случае однородной линейной системы двух линейных уравнений с двумя неизвестными: (рис.2.1)

29 Пример 7 Метод исключения Гаусса-Жордана (1/3)
Решите следующую однородную систему линейных уравнений, используя метод исключения Гаусса-Жордана. Решение Расширенная матрица Приведение этой матрицы к сокращенной форме строки-эшелона

30 Пример 7 Исключение Гаусса-Жордана (2/3)
Решение (продолжение) Соответствующая система уравнений Решение для главных переменных: Таким образом, общее решение: Примечание: тривиальное решение получается при s = t = 0.

31 год Example7 Исключение Гаусса-Жордана (3/3)
Два важных момента: ни одна из трех строковых операций не изменяет последний столбец с нулями, поэтому система уравнений, соответствующая приведенной строковой форме расширенной матрицы, также должна быть однородная система. Если данная однородная система имеет m уравнений от n неизвестных с m

32 Теорема 1.2.1. Однородная система линейных уравнений с большим количеством неизвестных, чем уравнений, имеет бесконечно много решений. Примечание: теорема применима только к однородной системе Пример 7 (3/3)

33 Компьютерное решение линейной системы
Большинство компьютерных алгоритмов для решения больших линейных систем основаны на исключении Гаусса или Гаусса-Жордана.Проблемы Уменьшение ошибок округления Минимизация использования пространства памяти компьютера Решение системы с максимальной скоростью

34 Набор упражнений 1.2, вопрос 11 Решите следующую систему методом исключения Гаусса.

35 год Набор упражнений 1.2, вопрос 17

36 Набор упражнений 1.2 Вопрос 20

37 Комплект упражнений 1.2, вопрос 31

38 Набор упражнений 1.2, вопрос 32


Онлайн-калькулятор обратной матрицы

Для любой неособой матрицы (т.е. определитель не равен нулю) существует обратная матрица , например, его произведение с исходной матрицей дает единичную матрицу:

А ∙ А −1 = А −1 ∙ А = E

Наш онлайн-калькулятор поддерживает два разных метода вычисления обратной матрицы: с помощью метода Гаусса-Жордана и с помощью составления алгебраических добавок к исходной матрице.

Чтобы найти обратную матрицу методом Гаусса-Жордана, нужно прикрепить единичную матрицу справа от исходной матрицы:

(A | E)

Затем с помощью элементарных преобразований преобразуйте исходную матрицу в единичную, применив те же преобразования к единичной матрице, записанной справа. Следовательно, исходная матрица будет преобразована в единичную, а выписанная справа единичная матрица — в обратную:

(A | E) → (E | A −1 )

Этот способ прост, удобен и не требует больших затрат времени.

Чтобы найти обратную матрицу с помощью метода алгебраических дополнений, можно использовать следующую формулу:

куда | А | — определитель матрицы А,
А и Дж — алгебраическое дополнение элемента и матрицы А.

По определению

A i j = (-1) i + j M i j

куда M и J — минор элемента а я j матрицы А.

По определению — второстепенный элемент а я j матрицы А — определитель, полученный удалением я строка, j столбец матрицы А.

Итак, метод алгебраического дополнения для нахождения обратной матрицы исходной матрицы порядка п очень трудоемко, потому что нужно вычислять не только определитель исходной матрицы, но и n 2 детерминанты порядка п-1 .

Страница не найдена | MIT

Перейти к содержанию ↓
  • Образование
  • Исследовательская работа
  • Инновации
  • Прием + помощь
  • Студенческая жизнь
  • Новости
  • Выпускников
  • О MIT
  • Подробнее ↓
    • Прием + помощь
    • Студенческая жизнь
    • Новости
    • Выпускников
    • О MIT
Меню ↓ Поиск Меню Ой, похоже, мы не смогли найти то, что вы искали!
Попробуйте поискать что-нибудь еще! Что вы ищете? Увидеть больше результатов

Предложения или отзывы?

Произошла ошибка при настройке вашего пользовательского файла cookie

Произошла ошибка при настройке вашего пользовательского файла cookie

Этот сайт использует файлы cookie для повышения производительности.Если ваш браузер не принимает файлы cookie, вы не можете просматривать этот сайт.

Настройка вашего браузера на прием файлов cookie

Существует множество причин, по которым cookie не может быть установлен правильно. Ниже приведены наиболее частые причины:

  • В вашем браузере отключены файлы cookie. Вам необходимо сбросить настройки своего браузера, чтобы он принимал файлы cookie, или чтобы спросить вас, хотите ли вы принимать файлы cookie.
  • Ваш браузер спрашивает вас, хотите ли вы принимать файлы cookie, и вы отказались.Чтобы принять файлы cookie с этого сайта, нажмите кнопку «Назад» и примите файлы cookie.
  • Ваш браузер не поддерживает файлы cookie. Если вы подозреваете это, попробуйте другой браузер.
  • Дата на вашем компьютере в прошлом. Если часы вашего компьютера показывают дату до 1 января 1970 г., браузер автоматически забудет файл cookie. Чтобы исправить это, установите правильное время и дату на своем компьютере.
  • Вы установили приложение, которое отслеживает или блокирует установку файлов cookie.Вы должны отключить приложение при входе в систему или проконсультироваться с системным администратором.

Почему этому сайту требуются файлы cookie?

Этот сайт использует файлы cookie для повышения производительности, запоминая, что вы вошли в систему, когда переходите со страницы на страницу. Чтобы предоставить доступ без файлов cookie потребует, чтобы сайт создавал новый сеанс для каждой посещаемой страницы, что замедляет работу системы до неприемлемого уровня.

Что сохраняется в файлах cookie?

Этот сайт не хранит ничего, кроме автоматически сгенерированного идентификатора сеанса в cookie; никакая другая информация не фиксируется.

Как правило, в cookie-файлах может храниться только информация, которую вы предоставляете, или выбор, который вы делаете при посещении веб-сайта.

Добавить комментарий

Ваш адрес email не будет опубликован. Обязательные поля помечены *